Pathophysiology Final Questions

Réussis tes devoirs et examens dès maintenant avec Quizwiz!

What is the highest risk factor for macular degeneration? a. Smoking c. Age b. Gender d. Race

c. Age

Small bowel obstructions lead to dilation of the intestine due to accumulation of: A. Blood and swallowed air B. WBC and air C. Swallowed air and GI secretions D. GI secretions and WBC

C. Swallowed air and GI secretions

The presence of normal endometrial mucosa abnormally implanted in locations other than the uterine cavity is known as: A. Placenta previa B. Ectopic dysmenorrhea C. Metaplastic coelomic epithelium D. Endometriosis

D. Endometriosis

A unilateral small kidney on ultrasound would suggest which of the following etiologies? a. Polycystic kidney disease d. Renal artery stenosis b. Hypertensive nephrosclerosis e. Malignancy c. Diabetic nephropathy

d. Renal artery stenosis

Which answer is a tumor suppressor gene associated with adrenal cancer, as well as many other types of cancer? A. T5 B. AA1 C. TP53 D. TP1

C. TP53

Hodgkin's lymphoma occurs due to a mutation of which of the following: A. B lymphocytes B. macrophage C. neutrophils D. T lymphocytes

A. B lymphocytes

Accumulation of what protein has been shown to be associated with dementia? A. β-amyloid protein B. Glycophorin D C. C-reactive protein D. Elastin

A. β-amyloid protein

Primary hyperparathyroidism is usually the result of: A. Carcinoma B. A single benign adenoma C. Leukocoria D. Hypocalcemia

B. A single benign adenoma

Which anemia is caused by antibodies attacking platelets, and presents with normal bone marrow? A. TTP B. ITP C. Factor V Leiden deficiency

B. ITP

What trait must a patient be homozygous for to have sickle cell disease? A. Hb F B. Hb A C. Hb S D. Hb C

C. Hb S

Excess of which of the following will cause nephrolithiasis? A. Calcium B. Uric Acid C. Oxalate D. all of the above

D. all of the above

A 45 year old female is in her second trimester of pregnancy and presents to her OB/GYN. The patient's age puts her at increased for complications. It was recommended that prenatal diagnosis to test for conditions in the fetus be performed. What diseases or conditions does prenatal diagnosis typically test for? a. Genetic disorders d. Spina bifida b. Down syndrome c. Cystic fibrosis e. All of the above

*e.All of the above*

A 45-year-old obese woman presents in the ER with steady, severe, aching pain in the upper right quadrant that radiates to the right scapula. The onset was acute and occurred 30 minutes after lunch. She had nausea with vomiting. On physical exam, she noted that she stops breathing on deep palpation of the right quadrant. Laboratory studies show an absolute neutrophillic leukocytosis with a left shift. What is top among your differential diagnosis? A. Acute Cholecystitis B. Abdominal Aortic Aneurysm C. Diverticulitis D. Colorectal Cancer

A. Acute Cholecystitis

All of the following may be presenting signs and symptoms of Pituitary Adenoma EXCEPT: A. Visual symptoms due to compression of optic nerve structures. B. Diplopia C. Ptosis D. Nonspecific headache E. All are possible presenting signs and symptoms of a Pituitary Adenoma.

E. All are possible presenting signs and symptoms of a Pituitary Adenoma.

A 35 y/o man who works in a chemical company comes in complaining about shortness of breath for past 3 days. He is febrile, has muscle aches and cough. Physical exam reveals yellow-green sputum production, wheezing, rhinorrhea, and adenopathy. His last physical exam was 2 months ago and his TB test was negative. What is the most likely diagnosis of this patient? A. Acute bronchitis B. Chronic bronchitis C. Tuberculosis D. None of the above

A. Acute bronchitis

If a patient had stomach cancer that began in the glandular cells that line the inside of the stomach and secrete a protective layer of mucus what kind of cancer would it be classified as? A. Adenocarcinoma B. Lymphoma C. Carcinoid D. Stromal

A. Adenocarcinoma

1 year old presents with splenomegally, jaundice, fatigue, and tachycardia. Blood work shows a hypochromic, microcytic RBCs with basophilic stippling. What is in your differential? A. Alpha Thalassemia B. Beta Thalassemia C. B12 deficiency D. Iron Deficiency E. A and B only

A. Alpha Thalassemia B. Beta Thalassemia *E. A and B only*

A 65y/o man comes into the office complaining of itchiness on his elbow. Upon examination you notice large blisters and urticarial lesions on his inner elbow. When you touch them, they do not rupture signifying a negative Nikolsky sign. You could treat this with all of the following EXCEPT: A. Antibiotics B. Corticosteroids C. Anti-inflammatories D. Immunosuppressants

A. Antibiotics

All of the following are TRUE except: A. Autoimmune thyroid is more frequent in Caucasians, than in Hispanics, Asians, or African Americans B. All thyroid diseases occur more frequently in women than in men. C. Autoimmune thyroid diseases have a peak incidence in people aged 20-40 years. D. All of the above are true

A. Autoimmune thyroid is more frequent in Caucasians, than in Hispanics, Asians, or African Americans

An intoxicated patient comes into the ER after being found down. You notice superficial lacerations to his forehead, arms and hands. The pt c/o diplopia and on physical examination you note significant swelling around the right orbit, proptosis, and most concerning-expulsion of intraocular contents. What is the most likely cause of his eye injuries and what emergent consult is needed? A. Blunt trauma, likely an assault. Ophthalmologist is needed to be consulted B. HTN emergency. Cardiology is needed to be consulted C. MVA. Vascular surgery is needed to be consulted D. Blunt trauma, likely an assault. Vascular surgery is needed to be consulted

A. Blunt trauma, likely an assault. Ophthalmologist is needed to be consulted

A 28 y/o female pt. presents to clinic with a red, swollen, tender area that is painful. The affected area seems to be located close to an open wound that the pt. received after she fell off her bike. On physical exam there seems to be sloughing skin and the redness seems to be spreading quickly. The patient complains of chills, fevers, and you notice there is gas in tissue and swollen lymph nodes. You determine the cause of the infection is Staph Aureus. What is most likely diagnosis? A. Cellulitis B. Candidiasis C. Molluscum contagiosum D. Lymphedema

A. Cellulitis

Which of the following does not cause chronic pancreatitis? A. Chronic IV drug use B. Autoimmune conditions C. Genetic mutations D. Blocked pancreatic duct or common bile duct E. Heavy alcohol use

A. Chronic IV drug use

A 55 y/o male comes into your ER at 10am with right upper abdominal pain. You smell alcohol on his breath. On PE you notice ascites and hepatomegaly. You check his PMH and find out that he has been to the ER before for chronic hepatitis. What is the most likely diagnosis? A. Cirrhosis of the liver B. GERD C. Appendicitis D. None of the above

A. Cirrhosis of the liver

A 68-year-old woman presents to the emergency department complaining of ear pain and purulent discharge. On otologic examination, the tympanic membrane is found to be intact. There is mild weakness of the ipsilateral face. Which of the following could predispose this patient to this condition? A. Diabetes Mellitus B. Hypertension C. Hypercholesterolemia D. Neurofibromatosis

A. Diabetes Mellitus

A mother brings in her 17 year old son for acute diarrhea that has lasted two days. She is doing her best to keep him hydrated with Gatorade and water, but she is concerned because he is constantly in the bathroom with watery stools about 6 times a day. You decide to draw labs, advise her to continue his fluid and electrolyte replacement at home, consider prescribing loperamide, and educate her that: A. Diarrhea is sometimes noninfectious, and due to an interruption in reabsorption of the 8-9L of fluid that enters the small intestines daily B. This is most like cholera and you must report it to the CDC immediately C. She should give her son an OTC cholinergic D. He must be a chronic drinker E. He should get plenty of rest and try to eat dense and heavy foods to help reabsorb the water

A. Diarrhea is sometimes noninfectious, and due to an interruption in reabsorption of the 8-9L of fluid that enters the small intestines daily

22 year old female presents with sore throat, fever, and a low-grade fever. She came in due to a rash that appeared yesterday on her hands and feet now spreading to her trunk. The lesions are symmetric, do not itch and have a violaceous center with pink halo separated by a pale ring. The patient did try on some new products at her store the other day. How would you classify her rash? A. Erythema Multiforme B. Hypersensitivity Type IV C. Hypersensitivity Type II D. A and C only E. A and B only

A. Erythema Multiforme B. Hypersensitivity Type IV *E. A and B only*

Which bones of the hand can tolerate a substantial degree of angulation without disability? A. First metacarpal B. Fourth metacarpal C. Fifth metacarpal D. All of the above

A. First metacarpal B. Fourth metacarpal C. Fifth metacarpal *D. All of the above*

29 year old female, 24 weeks gestation, presents to the OB/GYN clinic for a routine antenatal checkup. This is her second pregnancy, with her first one resulting in the birth of her first daughter. In her first pregnancy, she was diagnosed with gestational diabetes mellitus. Prior to both pregnancies, she was clinically obese. What routine test will be ordered on this visit to 55 screen for gestational diabetes mellitus? A. Glucose Challenge Test B. Glucose levels C. HbA1c levels D. Comprehensive Metabolic Panel E. Complete Blood Count

A. Glucose Challenge Test

Which strands of HPV account for the majority of genital warts? A. HPV 6 & 11 B. HPV 16 & 18 C. HPV 1 & 2 D. HPV 22 & 24

A. HPV 6 & 11

All of the flowing are associated with Huntington's disease EXCEPT: A. Huntington's disease is an inherited, X-linked recessive disorder B. Symptoms of the disease usually do not develop until after 30 years of age. C. The disease is characterized by progressive chorea and dementia; it is usually fatal within 15 to 20 years. D. Huntington's disease has no cure, and progression of the disease cannot be halted. E. All are associated with Huntington's disease

A. Huntington's disease is an inherited, X-linked recessive disorder

35 year old female presents to the ED with complaints of severe pain in the rectal area. She states that it started early yesterday morning and has not resolved since. No aggravating or alleviating factors. She reports recent onset of fever, for which she has been taking Tylenol for the rectal pain and fever. Vital Signs show Temperature 101.5, HR 72, RR 16, BP 110/80. Physical Exam shows anal mass at the 7 o'clock position consistent with a diagnosis of an Anal Abscess. What is the best treatment for her condition? A. Incision and Drainage B. Topical corticosteroid C. Fiber diet D. Watchful waiting E. Ciprofloxacin

A. Incision and Drainage

What is the most common cause of status epilepticus in children? A. Infections due to fever B. Trauma C. Changes or non compliance in medications D. Physical abuse

A. Infections due to fever

A patient's mother expresses that she is worried about her 16 year old daughter because she is irritable most of the day every day, has lost weight, rarely sleeps anymore, is frequently fatigued, and has lost interest in extracurricular activities. The mother states this has been going on for a month now. What diagnosis would you expect? A. Major Depressive Disorder B. None- Her behavior is normal for a teenager C. Chronic Fatigue Syndrome D. Anemia

A. Major Depressive Disorder

35 year old morbid obese female patient in the ER is presenting with nausea, abdominal pain, weakness, and fatigue. She is breathing heavy and has a PsaO2 of 93%. Upon asking questions you find she has a recent nuance of frequent urination, especially at night. She has been extremely thirsty and doesn't understand why. She came to the ER because she thinks she may be pregnant. She denies any hx of diabetes, CAD, stroke. After running tests, you find her bHCG is negative, but her serum glucose level is 650 and there are ketones in her UA. What diagnosis are you suspicious of? A. Metabolic acidosis B. Respiratory acidosis C. Respiratory alkalosis D. All of the above E. None of the above

A. Metabolic acidosis

If a 25 year old female presents to your office with dysfunctional uterine bleeding and she has received no prior treatments, what would you recommend? A. Oral contraceptives B. D/C C. Endometrial ablation D. Hysterectomy

A. Oral contraceptives

A 15 year old male complains of knee pain when running and jumping. The patient is on the high school track team. The patient has pain with knee extension, hamstring tightness, and an enlargement of the prominence of the tibial tubercle. Patient has full range of motion. What diagnosis would you suspect? A. Osgood- Schlatter Disease B. Sinding- Larson- Johansson Syndrome C. Perthes Disease D. Hoffa's Syndrome

A. Osgood- Schlatter Disease

Kyphosis is a curving of the spine that causes: A. Outward bowing/rounding of the back B. Sideways curvature of the spine C. Inward curvature of the lumbar and cervical spine

A. Outward bowing/rounding of the back

If a male patient is inable to retract the distal foreskin over the glans of the penis. What is this known as? A. Phimosis B. Paraphimosis C. Balanitis D. Cellulitis

A. Phimosis

The most common cause of Mitral Valve Stenosis is: A. Rheumatic heart fever B. Traumatic injury C. Genetic causes D. Marfan syndrome E. SLE

A. Rheumatic heart fever

35 year old male patient presents with pain and swelling in his left hand. He is a diabetic and has a serum glucose level of 350. He admits to not controlling it very well. Upon inspection of the hand you notice there is a small open sore, no bone or tendons can be seen. The patient has poor sensation in that area and does have some numbness. How would you classify his injury? A. Soft tissue injury of the hand B. Stage IV ulcer C. Burn D. Gangrene

A. Soft tissue injury of the hand

What is the most common pathogen that causes orbital cellulits? A. Staphylococcus aureus B. E. coli C. Batrachochytrium dendrobatidis D. Lactobacillus species

A. Staphylococcus aureus

54 year old asian-american male presents to the ED with diffuse, painful cutaneous rashes throughout his trunk and proximal extremities, which began 4 days ago. He states that he has no significant past medical history or allergies that he knows of. He was recently diagnosed with a UTI by his primary care physician, of which he was prescribed Bactrim (Sulfamethoxazole/Trimethoprim) 5 days ago. Physical exam shows necrotic, sheet-like loss of epidermis. Flaccid blisters are positive for Nikolski's Sign. What is the diagnosis? A. Steven Johnson Syndrome B. Urticaria C. Contact Dermatitis D. Eczema E. Psoriasis

A. Steven Johnson Syndrome

What finding is commonly found on a patient with rosacea? A. Telangiectasia of blood vessels B. Greasy skin C. Bruising of skin D. Warts

A. Telangiectasia of blood vessels

All of the following are true of orthostatic hypotension EXCEPT: A. There is a pooling of 200-300mL of blood to the lower extremities upon standing up. B. There is a transient decrease in blood pressure that cannot be compensated for. C. Depleted intravascular volume is a main cause. D. It refers to decrease blood pressure upon standing. E. None of the above.

A. There is a pooling of 200-300mL of blood to the lower extremities upon standing up.

Hemolysis can be due to hereditary or acquired disorders: A. True B. False

A. True

When evaluating someone with strabismus by using the Bruckner test, which eye will have a brighter/whiter reflex when light is shined towards the patient's eyes? A. Turned eye B. Normal eye C. Non turned eye

A. Turned eye

A mother brings her Caucasian 11-year-old daughter in to the ED because she has not been herself lately. The mother states that she has noticed her daughter is wetting the bed lately and is drinking a lot more than she used to. When you exam the little girl, you notice that she is taking rapid and deep breaths. Her vital signs show hypotension and you suspect she is dehydrated. You try to ask the girl how she is feeling but she is not responding well. Based on the history from the mother and your findings during the physical exam, what is the most likely cause of her symptoms? A. Type 1 DM B. Type 2 DM C. Lead Nephropathy D. UTI

A. Type 1 DM

Patient is a 25 year old male presenting to the clinic with nocturia, urgency, painful ejaculation, pain in the abdomen and groin and a mild fever. A digital rectal exam is performed and mild enlargement is noted. What is the most likely cause of this pathology? A. gonorrhoeae and Chlamydia trachomatis B. HIV C. C. trachomatis D. Trichomonas vaginalis E. E. coli

A. gonorrhoeae and Chlamydia trachomatis

Kaposi sarcoma (KS) is caused by infection with the virus: A. human herpes virus 8 (HHV8) B. cytomegalovirus (CMV) C. herpes zoster virus D. hepatitis B (HBV)

A. human herpes virus 8 (HHV8)

Bulimia nervosa is an eating disorder defined by A. secretly binging — eating large amounts of food — and then purging, trying to get rid of the extra calories in an unhealthy way. B. an abnormally low body weight, intense fear of gaining weight, and a distorted perception of body weight. C. consuming very large quantities of food in a brief period and feeling out of control during the binge, but not trying to rid of the excess calories. D. none of the above

A. secretly binging — eating large amounts of food — and then purging, trying to get rid of the extra calories in an unhealthy way.

Who has the highest incidence of cystitis? A. Sexually active elderly women B. Sexually active men age 20-45 C. Sexually active women age 20-45 D. Impotent elderly men E. Newborns

C. Sexually active women age 20-45

A 40-year-old Caucasian male visits your office complaining of lower back pain that is keeping him awake at night. He works in the warehouse of a factory and does a lot of lifting on a daily basis. He cannot recall a specific event that led to his pain, but claims that it has gradually increased over time. You inform him that lower back pain is a common complaint among American's, even at his age. You educate him that all of the following can be causes of lower back pain: a. Muscle or ligament strain d. Spinal stenosis b. Osteoporosis e. All of the above

All of the above

Decreased platelet production may be observed in which of the following conditions? a. Hypersplenism d. Alcoholism b. DIC e. Aplastic anemia c. Henoch-schonlein disease

Aplastic Anemia

You are examining a 22 year old male patient's scalp during their annual physical and you notice round patches of hair loss. You don't notice any scars. What diagnosis would you suspect? A. Tinea capitis B. Alopecia C. Telogen effluvium D. Tricholtillomania

B. Alopecia

Upon examination you find that the patient is positive for De Musset sign, Quincke sign, and Traube Sign. What should your number one differential diagnosis be? A. Pulmonic Regurgitation B. Aortic Regurgitation C. Pulmonic Stenosis D. Aortic Stenosis

B. Aortic Regurgitation

Patient is a 23-year-old African-American female who is presenting to the OB/GYN clinic complaining of an abnormal fishy smelling discharge and recent pain during intercourse. The odor is more prominent after intercourse. She has noticed changes in the color of her discharge and states that it is grayish, foul-smelling discharge. She has reported recently sleeping with 5 new partners within the last month. Based on the presenting symptoms, what is most likely the patient's diagnosis? A. Yeast infection B. Bacterial vaginosis C. Trichomoniasis D. Non-infectious vaginitis E. All of the above are likely to causing the patient's presenting symptoms

B. Bacterial vaginosis

The following definition describes which type of psychological disorder?You've had at least one manic episode. The manic episode may be preceded by or followed by hypomanic or major depressive episodes. Mania symptoms cause significant impairment in your life and may require hospitalization or trigger a break from reality (psychosis). A. Depression B. Bipolar Disorder C. Schizophrenia D. Delusional Disorder

B. Bipolar Disorder

Mrs. Chang is a 48-year-old woman who has cor pulmonale. Additionally she is experiencing a cough that she describes has a mucupurulent cough lasting for the past two years. She recently noticed some blood in her sputum and began to be a little worried. While performing her physical exam, you note that since her last visit, she has lost fifteen pounds and has clubbing around her fingers. What is her diagnosis? A. Bronchitis B. Bronchiectasis C. Pneumonia D. Fungal pneumonia

B. Bronchiectasis

Frank comes to the office today for a routine physical as he has done for the past 6 years. There has been no complaints in the past and seems to be a normal healthy white male at the age of 65. While doing the PE, you note that he has enlarged painless lymph nodes and can palpate his spleen. When asked if anything has changed, he explains that even though his wife has hot flashes and sleeps with air on 70 degrees F, He wakes up sweating. He has lost 10 pounds in the past 2 months. You order a CBC and it returns with a high level of B Cells. What is the diagnosis? A. aplastic anemia B. CLL. (Chronic lymphocytic leukemia) C. Sickle cell Anemia D. G6PD

B. CLL. (Chronic lymphocytic leukemia)

A mother comes into the ED with her 6 month old infant who has Down Syndrome. The infant presents with: Fast breathing/difficultly breathing Poor feeding habits Rapid pulse Shortness of breath Sweating while feeding Tiring very easily Poor growth The mom complains that the baby is quite irritable and has had multiple nose bleeds. What would be the best diagnostic test to run first? A. CBC for infection B. Chest x-ray C. Chromosomal Karyotype D. No further testing needed, start augmentin ASAP

B. Chest x-ray

A 32-year-old Hispanic pregnant mother is present at the clinic today with a chief complaint of sudden, intensifying pain in her right upper quadrant that began 4 hours ago. She noticed the pain this morning while doing chores around the house. She is 24 weeks pregnant and has no significant PMH. Vital signs are all WNL. According to the presented circumstances, what is the most likely differential diagnosis? A. Appendicitis B. Cholelithiasis (gallstones) C. GERD D. Cirrhosis

B. Cholelithiasis (gallstones)

Asthma is a mainly a result of: A. Lung cancer B. Chronic, reversible inflammation C. Chronic, irreversible inflammation D. I need an inhaler because of this question

B. Chronic, reversible inflammation

2 year old white male infant is presenting with recurrent respiratory infections. The orphanage thinks he has failure to thrive even though has a good appetite. You run labs and find the patient has a higher than normal salt content in his sweat. What are you suspicious the patient has? A. HIV B. Cystic Fibrosis C. Atypical asthma D. Lung cancer

B. Cystic Fibrosis

The force of blood pumping can split the layers of the artery wall, allowing blood to leak in between them. This process is termed a: A. Aortic aneurysm B. Dissection C. Embolism D. Thrombus

B. Dissection

A 28-year-old male presents with pain and tenderness of his right testicle. These symptoms started gradually over the last day. On physical exam, his pain is lessened with elevation of the effected testicle. What is most likely? A. Testicular Torsion B. Epididymitis C. Testicular Cancer D. Trauma

B. Epididymitis

A 42 year old male patient comes into your office complaining of burning pain above his abdomen, bloating, and vomiting. During his history he admits to excessive drinking, smoking and being very stressed at work. He also takes ibuprofen regularly for pain in his knee. What is the most likely diagnoses? A. Crohn's Disease B. Gastritis C. Hiatal Hernia D. GERD

B. Gastritis

A 5-mo old male with no pertinent past medical history, presented with a scrotal mass in the right testicle of 4 to 6 wk duration. The scrotal mass measured 3 x 5 cm, was non tender, and unilateral. The mass appeared cystic and communicated, and upon physical examination there was ability of getting fingers above the mass, and transillumination was positive. What is your diagnosis? A. Varicocele B. Hydrocele C. Testicular torsion D. Epididymitis E. Prostatitis

B. Hydrocele

A new mother has noticed her 7mo old son was becoming irritable and has had some unexplained bruising on his arms and legs. She realizes babies get bumps and bruises, but she is worried about how frequently they occurring. He also cries in pain when trying to crawl and walk. When she noticed blood on his teething toys she made a doctors appointment. Upon questioning, the PA discovered that after his circumcision, he bled for a few days. This led the PA to believe the baby had a coagulation disorder. Mom also revealed there was a family history of bleeding disorders on her side. After ordering a CBC, CMP, PT and PTT the PA notices one abnormal value. Which value is it? What value would you be most interested in seeing next? A. Increased Reticulocytes/Serum Iron B. Increased PTT/Assay of Factor VIII:C C. Increased PTT/Factor IX Assay D. Decreased Reticulocytes/Serum B12

B. Increased PTT/Assay of Factor VIII:C

A 45 y/o man comes to the ER after being in a car accident. He seems to have no major injuries, however he is complaining of chest pain and as you make your visual observation, you notice he is diaphoretic, and showing signs of dyspnea and mild cyanosis. You check his vitals and they are as follows: Pulse 110, BP 95/65, Respirations 25/min. CXR is abnormal. How would you proceed? A. Consult a Thoracic surgeon B. Insert a Chest Tube C. Order an Albuterol treatment D. Start an antibiotic

B. Insert a Chest Tube

The accumulation of protein-rich fluid in the interstitial spaces that is caused by a disruption or obstruction of lymphatic flow and drainage is known as A. Lymphadenopathy B. Lymphedema C. Elephantitis D. Lipedema E. Deep vein thrombosis

B. Lymphedema

A mother brings her 9 year old son into the ER after he sustained an injury while playing soccer. You note swelling and tenderness on palpation of the ankle. The pt is crying, stating his whole foot hurts him. You order an X-ray but being the astute PA that you are, you already believe the pt has what type of injury as it is the most common? A. Calcaneous fracture B. Malleolar fracture C. Distal femur fracture D. Colles fracture

B. Malleolar fracture

A 24 y/o female presents complaining of light headedness and dizziness upon exertion. Vital signs are within normal range and she has no pertinent medical history. Auscultation of the heart reveals a mid systolic click heard over the apex. What would be your number one differential diagnosis? A. Aortic Stenosis B. Mitral Valve Prolapse C. Cardiac Tamponade D. Pericarditis

B. Mitral Valve Prolapse

63 year old male presents to the ER with chest pain and a past medical history of hypertension. He is complaining of shortness of breath and nausea. He is diaphoretic with a BP of 92/65, HR of 65, with PsaO2 of 98%. You order an EKG, which shows ST depressions. Troponin, CKMB, CK, and BNP labs are still pending. You consult a cardiologist because you are suspicious of? a. STEMI d. Coarctation of aorta b. NSTEMI e. CHF c. Food poisoning

B. NSTEMI

All of the following can be seen in a patient with Open-Angle Glaucoma except: A. Optic neuropathy that has a progressive loss of peripheral visual field followed by central field loss which usually but not always occurs in the presence of elevated intraocular pressure (IOP). B. Narrowing or closure of the anterior chamber angle. C. Optic nerve or "disc" appears hollowed-out on ophthalmoscopic examination, this is referred to as "cupping." D. Open-angle glaucoma, optic nerve damage caused a progressive loss of retinal ganglion cell axons, which initially presents as a visual field loss and followed ultimately by irreversible blindness if left untreated.

B. Narrowing or closure of the anterior chamber angle.

For Lent this year, Ms. Lily decided that she would abstain from all forms of meat. For the past two weeks, she has been feeling slight fatigue, dizziness, and headache that comes and goes for the past two weeks. She is not currently taking any medications and has no known allergies. Upon examination, you notice angular stomatitis and glossitis. Based on this history, how would you treat Ms. Lily? A. Blood transfusion B. Oral iron therapy C. Oral cobalamine D. Splenectomy

B. Oral iron therapy

A 67 y/o obese, white, female comes to you complaining about pain in her knee joint. She says she has a grating sensation in her knee and has loss of flexibility. Pt. has family history of osteoarthritis and has a personal hx of diabetes. Join fluid analysis helps you r/o gout. What is the most likely diagnosis of this pt.? A. Juvenile rheumatoid arthritis B. Osteoarthritis C. Reiter syndrome D. Septic arthritis

B. Osteoarthritis

All of the following are indicative of a patient with unstable angina pectoris EXCEPT: A. Tachycardia B. Pain in gastrocnemius C. Bradycardia D. Diaphoresis

B. Pain in gastrocnemius

A six-year-old boy comes into the clinic complaining of an itchy and crusty eye. He tells you that he noticed it starting itching two days ago while at school. He explains that when he wakes up in the morning he has "gross, crusty stuff on the edges." On inspection of his right eye, you notice the conjunctiva is red with a yellow colored discharge. The left eye is normal. No abnormalities noted on fundoscopic examination. You diagnose this patient and determine a treatment plan. What would be considered to be the most appropriate plan of treatment? A. Leave untreated and assure patient that it should resolve within 2-3 weeks B. Prescribe antimicrobial eyedrops C. Explain to patient that his allergies exacerbated his symptoms and recommend OTC antihistamines D. No treatment required, just follow up in 2 weeks to confirm resolution of symptoms E. I don't want to be a PA anyway

B. Prescribe antimicrobial eyedrops

What causes most cases of pulmonary valve incompetence? A. Atherosclerosis B. Pulmonary HTN C. Pneumonia D. Acute Rheumatic Fever E. Natural process of aging

B. Pulmonary HTN

Which of the following is not a common cause of Croup? A. Parainfluenza virus B. Adenovirus C. Streptococci D. Respiratory syncytial virus (RSV) E. Enterovirus

C. Streptococci

All of the following are false regarding Respiratory Syncytial Virus EXCEPT: A. Respiratory syncytial virus (RSV) is membrane-bound deoxy- ribonucleic acid (DNA) virus that causes acute respiratory tract illness in all age B. RSV-induced bronchiolitis causes acute inflammation, edema, and necrosis of the epithelial cells lining the small airways, air trapping, bronchospasm, and increased mucus production C. Infection spreads via an infected animal vector or via blood borne transmission D. All are false regarding Respiratory Syncytial Virus E. All are true regarding Respiratory Syncytial Virus

B. RSV-induced bronchiolitis causes acute inflammation, edema, and necrosis of the epithelial cells lining the small airways, air trapping, bronchospasm, and increased mucus production

22 year old female presents to the ED with a recent onset of deep, rapid breathing that she states began 1 day ago while she was studying for 3 exams at the end of the week. She is a PA student at a local university and states that these attacks occur quite often, especially during big test weeks. You order a CXR, CBC, CMP, TSH, ABG, and Urine Toxicology test. All return negative except for the ABG, which shows a PaCO2 of 25 (low). You instruct her to breathing into a paper bag and she starts to calm down instantaneously. What was this patient's most likely primary acid-base disturbance upon arrival to the ED? A. Respiratory Acidosis B. Respiratory Alkalosis C. Metabolic Acidosis D. Metabolic Alkalosis E. Mixed Acid-Base Disorder

B. Respiratory Alkalosis

Romana comes into your office for a monthly prenatal check up. Last month you drew labs to determine her blood type for any unexpected complications during labor. The results indicated that her blood type was AB negative. What is the suggestion of the provider? A. there is no treatment needed B. RhoGAM before and 72 hours after labor C. Amniocentesis D. Ultrasound

B. RhoGAM before and 72 hours after labor

A 22 year old women presents to the ER with a 3 mm laceration over the occipital area of her skull. Bruising is seen on both wrists and on the left shoulder. The patient claims to have tripped while walking down the stairs. The patient lives with her boyfriend and their one year old baby. What should always be considered in a situation such as this? A. A neurological disorder, which would explain her falling down the stairs B. The possibility of domestic abuse C. The patient is very clumsy and needs to be more careful walking on the stairs D. Mitral Valve prolapse may have caused her to to faint and fall down the stairs

B. The possibility of domestic abuse

Which of the following is not a sign or symptom of substance abuse? A. Over time, needing more of the drug to get the same effect B. Understanding the addiction and preventing relapse during treatment C. Driving or doing other risky activities D. Cutting back on social or recreational activities

B. Understanding the addiction and preventing relapse during treatment

The most common cause of encephalitis is: A. Bacterial B. Viral C. Fungal D. Arthropod

B. Viral

If a patient presents with infectious diarrhea, what should you look for in their stool? A. Hair B. WBC C. RBC D. platelets

B. WBC

All of the following are medications to treat PREeclampsia EXCEPT: A. Benazepril B. Methyldopa C. Betamethasone D. Magnesium Sulfate E. All of the above medications can be used to treat preeclampsia

Benazepril

Which of the following diseases would most likely cause aortic stenosis? A) Mononucleosis B) Measles C) Rheumatic Fever D) HIV

C) Rheumatic Fever

15 year old male presents with temperature of 101 F, chills, sore throat, and a hoarse voice. Upon inspection, his tonsils are of normal size, no exudates, or redness are noted in the throat. No tenderness in the sinuses. Lung and chest are normal. His deep cervical lymph nodes are palpable. The thyroid feels enlarged upon palpation, and the patient is complaining of tenderness. What are you suspicious the patient may be presenting with? A. Goiter B. Grave's disease C. Acute Thyroiditis E. Hyperthyroidism

C. Acute Thyroiditis

What is the most common type of pancreatic cancer? A. Giant cell carcinoma B. Adenosquamous carcinoma C. Adenocarcinoma of the ductal epithelium D. Type 1 carcinoma

C. Adenocarcinoma of the ductal epithelium

A patient is brought in by ambulance because they are experiencing delirium tremens. What do you suspect? A. Opioid Withdrawal B. Stimulant Withdrawal C. Alcohol Withdrawal D. An overdose

C. Alcohol Withdrawal

A 50-year old male presents to your office complaining of an intense pain in his left great toe for the last 5 hours. He has no outstanding past medical history except drinking heavily on a regular basis for the past 26 years. He denies any trauma to the area. Upon examination the area around his great toe is slightly red and the range of motion is decreased. He has never had this pain before but rates his pain a 9 out of 10 with the worst pain he ever felt being post-operative pain. You believe this is gout, what is this associated with? A. Underexcretion of uric acid B. Overproduction of uric acid C. All of the above D. None of the above

C. All of the above

All of the following are causes of post partum hemorrhage EXCEPT: A. The failure of the uterus to contract and retract following delivery B. Large birth weight causing vaginal trauma C. An abnormally high maternal platelet count D. Thrombosis E. Previous postpartum hemorrhage

C. An abnormally high maternal platelet count

A 16 year old male presented to the ER complaining of pain in his shoulder, the patient stated that during a recent basketball game while blocking a shot fell and hurt his shoulder, he demonstrated a fall on an abducted, externally rotated and extended arm. What kind of injury do you suspect and what deformities is it associated with? A. Posterior shoulder dislocation with bankart lesion B. Inferior shoulder dislocation with Hill Sachs deformity C. Anterior shoulder dislocation with Hill Sachs deformity and Bankart lesion D. None of the Above

C. Anterior shoulder dislocation with Hill Sachs deformity and Bankart lesion

Mrs. Jones brings her son into your office. She is frustrated that he is getting in trouble in school and his grades are declining. At home Jonny is not listening and tends to not finish things that he starts. She is concerned there might be an issue nice his Grandfather had similar issues and it resulted in many failures to achieve his dreams. Johnny is unorganized and has a difficult focusing. All of the following are likely diagnosis except? A. ADHD B. ADD C. Bipolar disorder D. Hyperactive disorder

C. Bipolar disorder

Which of the following is the last resort treatment for gum disease? A. Deep cleaning B. Antimicrobial mouthrinse C. Bone/tissue graft D. Antiseptic chips E. Scaling and root planning

C. Bone/tissue graft

Which of the following does NOT put a woman at risk for fibrocystic breasts? A. Nulliparity B. Late menopause C. Breast cancer D. Use of estrogen-replacement therapy

C. Breast cancer

A 33-year old woman presents to your office complaining of a cottage cheese-like vaginal discharge and a burning sensation when she urinates and during sexual intercourse. After taking her history you discover she got divorced a year ago and since then has had multiple partners. What disease does she most likely have? A. Bacterial vaginosis B. Trichomonas infection C. Candidiasis D. Herpes Simplex Virus

C. Candidiasis

A 42 y/o man comes to you with a complaint about a lump under his skin in the neck and left armpit. He is worried about his unintended weight loss and he tested HIV positive 1 year ago. On physical exam you find he has enlarged spleen and lymph nodes. What is the likely diagnosis? A. Cellulitis B. Candidiasis C. Castleman disease D. Gout

C. Castleman disease

A 27 year old female comes into the emergency room on crutches with extreme pain. She's having trouble saying what's wrong due to the pain but her boyfriend states she fractured her tibia two days ago from a skiing accident. She describes her pain finally as a 12 out of 10 and describes it as burning and deep. Her toes are numb. What is the most likely diagnosis? A. She's just a baby B. She re-fractured her leg C. Compartment syndrome D. Shin splints

C. Compartment syndrome

A 35 year old female patient who gave birth 3 months ago complains that she feels like "something is falling out of my vagina." The patient admits to repeated bladder infections and feeling increased discomfort when bearing down. What diagnosis do you suspect? A. This is a normal feeling up to 6 months after giving birth. B. Rectocele C. Cytocele D. UTI

C. Cytocele

A 42-year old male presents with shortness of breath after walking up the stairs and continued after laying down. After a thorough exam you discover some edema, rapid heart rate, an S3 gallop, and a displaced maximal impulse. You catch a glimpse of his fingers and see some clubbing and cyanosis. You believe his condition is idiopathic and the third most common cause of heart failure, what is it? A. Hypertrophic cardiomyopathy B. Restrictive cardiomyopathy C. Dilated cardiomyopathy D. Angina pectoris

C. Dilated cardiomyopathy

All are acceptable treatments for generalized anxiety disorders EXCEPT: A. SSRIs B. CBT C. Electrotherapy D. Hospitalization E. A and B only

C. Electrotherapy

What is a disorder characterized by a widespread of chronic musculoskeletal pain as well as a heightened response to painful pressure who's etiology is not fully understood? A. Muscular Dystrophy B. Atrophic Lateral Sclerosis C. Fibromyalgia D. Multiple Sclerosis

C. Fibromyalgia

What should you counsel a new mother with mastitis to do? A. Stop breastfeeding B. Feed the baby more frequently C. Fully empty breasts during feedings D. Corticosteroids on affected area E. Give the baby antibiotics so you don't infect him

C. Fully empty breasts during feedings

A 52 year old male reports to your office for complaints of a cough that won't go away, chest pain from the cough, and difficulty breathing. Your history reveals that he is a farmer in the Mississippi River Valley, known to be heavily populated with bats. He was diagnosed with SLE when he was 17 years old. Your physical exam reveals a fever, tachycardia, tachypnea, and rales bilaterally. You order a CXR and see bilateral consolidation in the lungs. With the information at hand, what is the most LIKELY diagnosis of your patient? A. Influenza B. COPD C. Fungal Pneumonia D. Viral Pneumonia

C. Fungal Pneumonia

A 15 year old female presents to your office complaining of a sore throat, headache, and malaise. On physical exam you notice tonsillar exudation, a "hot potato-sounding" voice, erythema, and asymmetry of the soft palate. Upon further questioning you discover she had an acute tonsillitis infection a few weeks ago. You believe this to be a peritonsillar abscess. Which of the following is the most common cause? A. Lactobacilli B. Epstein-Barr virus C. Group A beta-hemolytic streptococci D. Haemophilus

C. Group A beta-hemolytic streptococci

Which of the following is NOT a cause of Cerebral Palsy? A. Idiopathic B. Pre-term birth C. Head injury D. Prenatal factors

C. Head injury

A 26 year old female presents in your office complaining of N/V x 3 days and rectal bleeding x 1 day. PMH of IBS and just had a vaginal birth 5 weeks ago. She is concerned because she has never seen the blood on the toilet paper before. She also admits to blood, in toilet, anal itchiness, and claims there is a small bump on her anus. She denies pain on defication. What is your diagnosis? A. Anal fissure B. Rectal Prolapse C. Hemorrhoid D. Fistula

C. Hemorrhoid

A long distance runner complains of a general dull pain in his foot while running. He claims there was no trauma to the foot. He has just got done running his twentieth marathon last week. You decide to order an x ray of his foot. Which type of fracture would be most likely expect to see on x ray? A. Comminuted Fracture B. Displaced Fracture C. Stress Fracture D. Open Fracture

C. Stress Fracture

You are working the late shift on a cold night in the ED. A local 40 y/o indigent female is brought in as is the norma;l to get out of the cold. Also as usual you had to set up her cat in a carrier outside of the ED. She does not appear in distress and smells aweful, but because you are a dedicated professional you do a thorough exam on her. You notice pea-sized to marble-sized lumps under her skin under her breasts. All lymph node are nonpalpable. Some of the lumps appear broken open and a foul-smelling pus. Pus exudate surrounds the unclean area. Upon light palpation the patient expresses pain 6/10. Upon futher inspection you find several more lumps between her buttocks as well. When you take her PMH she says she has had those lumps for many years. What do you suspect for a diagnosis: A. Scabies B. Contact dermatitis C. Hidradentitis suppurativa D. Cat Scratch Disease (CSD)

C. Hidradentitis suppurativa

The female mite burrows just beneath your skin and produces a tunnel in which it deposits eggs. What causes the intense itching? A. the eggs hatching B. healing process C. Histamine released from cells D. the female laying eggs

C. Histamine released from cells

A 16 y/o diabetic male, having just flown in from Texas is on the first day of a school band trip is trying out a new snow board park at a major ski resort. His companions lose sight of him at the entrance to the park but eventually find him at edge of the park sitting in the snow at the foot of the 3rd 50' table top jump. He appears agitated but fine. He is not bleeding. He denies any sprains or pain. He is wearing a helmet. He thinks he may have crashed when he landed but he isn't sure. His friends remain concerned and call ski patrol. Based on how he presents why would he require advanced care? A. DVT B. Pathologic Fracture C. Intracranial Hemmorhage D. DKA

C. Intracranial Hemmorhage

A pregnant women presents with patches of hyperpigmentation on her face. What do you suspect? A. Acanthosis Nigricans B. Contact Dermatitis C. Melasma D. Drug-Induced Photosensitivity

C. Melasma

Mr. Smith is a 30 year old male who has come to your office complaining of morning headaches and dry mouth over the last year. He has had difficulty concentrating during the day and has been irritable. You do a physical exam find that his tonsils are large without erythema and all his vitals were normal. He states in his history that he smokes regularly and drinks during the weekend. What is the mostly likely cause for his complaints? A. Eptstein Barr Virus B. Asthma C. Obstructive sleep apnea D. Hyperthyroidism

C. Obstructive sleep apnea

78 year old female presents to the clinic stating she is feeling tired for the past 2 months. She also reports that she is feeling tingling sensations in her toes at random times throughout the day. Her daughter, who is with her at the office, states that she has started a vegan diet 6 months ago in an attempt to lose weight and live a healthier lifestyle. Physical exam reveals difficulty balancing and mild ataxia. CBC is ordered and shows anemia with an MCV of 115. What is the proper treatment for this patient? A. Folic Acid B. Iron tablets C. Oral Vitamin B12 formulation D. Calcium supplements E. Niacin

C. Oral Vitamin B12 formulation

Meniere disease presents with all of the following except: A. Vertigo B. Hearing loss C. Pain in the inner ear D. Tinnitis E. All of the above are found in Meniere disease

C. Pain in the inner ear

A very well dressed pt comes into your clinic. She states she has pain in her index finger. You learn from taking the history that the pt receives regular manicures and pedicures. Upon visualization, you note the distal metaphalange on her second right finger is swollen with noted pus, erythematous and warm to touch at the nail fold. Pt is afebrile with a normal WBC. You know the most likely causative agent is Staphylococcus aureus but may also be from Pseudomonas as the patient has recently had a manicure. Based off the clinical appearance, what is the likely diagnosis and how do you treat? A. Paronychia; IV Vancomycin for a week B. Squamous cell carcinoma; excision of abscess and ensure clear margins C. Paronychia; incision and drainage; warm compresses/soaking at home D. Verrucea; cryotherapy

C. Paronychia; incision and drainage; warm compresses/soaking at home

A 17 year old wrestler comes into the office with a large auricular hematoma. He states that he obtained the hematoma 4 days ago during one of his matches and it does not seem to be resolving on its own. The hematoma is measured to be greater than 2cm. What is the best course of treatment? A. Give it time. It will eventually resolve on its own B. Perform needle aspiration C. Perform incision and drainage D. Prescribe an anticoagulant

C. Perform incision and drainage

Jamie is a 12 year old girl who has been experiencing a sore throat and fever. She states that she has been feeling very itchy around her abdominal area. Upon examination, you notice a Herald patch along with small scaly spots that move across the back and chest, almost resembling a "Christmas tree" pattern. What is your diagnosis? A. Chicken Pox B. Measles C. Pityriasis Rosea D. Parvovirus B19

C. Pityriasis Rosea

Where is the most common place a cerebral aneurysm occurs? A.Anterior circulation B. Lateral circulation C. Posterior circulation D. they occur evenly in each of the above.

C. Posterior circulation

A 12-year-old boy presents to the office with an injury from football. He states that he "twisted his ankle and hurt it real bad." Upon examination you note a mildly swollen left ankle with limited ROM. X-rays are unremarkable and show no fractures or anatomical displacement. You diagnose him with a left mild sprain on the ankle and recommend him to treat this by...? A. Surgery to repair the ankle sprain B. Cast for 6 weeks C. RICE- rest, ice, compression and elevation of ankle D. Heavy doses of pain meds and bed rest for 6 weeks

C. RICE- rest, ice, compression and elevation of ankle

A 47-year-old man who is HIV negative presents with a nonproductive cough and SOB. Physical examination reveals +bronchophony, +whispered pectoriloquy and rales on his chest examination. His diagnosis is likely: A. Adenovirus B. Bordetella Persussis C. Respiratory syncytial virus D. Pneumocystis

C. Respiratory syncytial virus

A 72 year old Hispanic male came into the ER with rapid breathing, shortness of breath, and tachycardic. He has a history of diabetes mellitus II. His father and grandfather have a history of lung cancer. Upon examination, you notice that his skin appears cyanotic, with pulse racing, and crackles in his lungs. You conclude that this patient is in cardiogenic shock and is not receiving enough blood to meet his body's needs. What would be the next best step in treating this patient? A. Placement of central line B. Take patient to the OR to perform a CABG C. Resuscitative therapy D. Placement of pulmonary artery catheter

C. Resuscitative therapy

Which of the following is FALSE regarding acute pharyngitis? A. Group A streptococcus is most prevalent during late fall through early spring. B. Other symptoms that may be associated with acute pharyngitis are cough, fever, and tonsillar exudate. C. The most common bacterial cause is staphylococcus aureus. D. Coronavirus can cause irritation of pharyngeal mucosa secondary to nasal secretions.

C. The most common bacterial cause is staphylococcus aureus.

Upon pelvic exam, a bulge is seen of the posterior wall of the vagina when the patient bears down. What must be considered? A. Rectal Prolapse B. An STD C. Vaginal Prolapse D. Pregnancy

C. Vaginal Prolapse

As it pertains to pressure ulcers, which of the following is not a treatment for this condition? A. First and foremost, patients who are at risk or who have active ulcers must be turned by professional care givers at least every two hours. The use of pillows, wedges and other positioning devices are encouraged. B. Wound care consult from either a trained nurse or physician can be key to gain insight into offloading weight and also to develop a plan of attack for wound healing C. Vitamin B4 D. Treatment for infection is indicated only if signs warrant. Many pressure ulcers never get infected and treatment should be based on empirical evidence of infection

C. Vitamin B4

While performing a Caesarean section, a single-layer uterine closure is used when: A. The pregnancy is multiple gestation B. There was complications during the surgery C. When the mother does not want a future pregnancy D. The surgeon is running late and wants to eat his corn beef on rye

C. When the mother does not want a future pregnancy

A 75 y/o woman presents to ED after falling in her home. She tripped on a rug and landed with her arms extended and her hands outstretched. She presents complaining of left wrist pain. Radiographs reveal a dorsally angulated and displaced distal radius metaphyseal fracture. What is the most likely diagnosis? a. Barton fracture c. Smith fracture b. Colles fracture d. boxer fracture

Colles Fracture

Which is a sign of conduct disorder in children? A. Stealing B. Fire setting C. Minimal appetite D. A & B E. All of the above are signs

D. A & B

Joe has just returned from a scuba diving vacation in Fiji where he was doing multiple 150-foot dives. He noticed mild ringing and hearing loss in his ear this past week upon returning from his vacation that precipitated his trip to the office today. He has noticed no discharge and reports feeling no pain. Due to the mechanism of injury and the presenting symptoms, on otoscopic examination you would expect to see.... A. A bulging errymatous tympanic membrane with purulent discharge B. A sea urchin C. Impacted cerumen D. A perforated ear drum

D. A perforated ear drum

An 18 year old female presents to your office explaining that she hasn't been feeling like herself lately. She used to have good grades but hasn't been able to focus in school and her grades are falling. She feels constantly anxious, sad and worried but she doesn't know why. She has been drinking more than usual on the weekends and got so drunk at a party last weekend her boyfriend broke up with her, claiming she hasn't been the same since her parents divorce 6 months ago. You are beginning to think your patient may be suffering from: A. Major Depressive Disorder B. Histrionic Personality Disorder C. Antisocial Disorder D. Adjustment Disorder E. PMS

D. Adjustment Disorder

54 year old male comes in complaining of terrible headaches that keep waking him up in the middle of the night. There always seems to be intense pain on the right side of his head by his eye when it happens. He states the headaches will occur for about 15 days straight then go away for awhile. You diagnose your patient with cluster headaches. What type of preventative/prophylactic treatment could you prescribe your patient? A. Mood stabilizers B. CCBs C. Anticonvulsants D. All of the above E. None of the above

D. All of the above

All of the following are risk factors associated with oral candidiasis except? A. Immune compromised B. Wearing dentures C. Oral corticosteroids D. All of the above

D. All of the above

Hypervolemia can result from: A. CHF B. Kidney failure C. Liver failure D. All of the above

D. All of the above

Which of the following is/are example(s) of hypertensive emergency pathophysiology? a. Abrupt increase in systemic vascular resistance b. Endothelial injury c. Deposits of platelets and fibrin d. All of the above

D. All of the above

6 year old patient presented today with intense pruritus causing scratching and the development of a rash. The patient was afebrile and is currently only on medication for asthma and seasonal allergies. All of the following are true regarding the condition described except: A. Atopy is one of the most common inflammatory skin disorders, it is considered apart of the triad of atopy which includes asthma and allergic rhinitis. B. Atopic skin has decreased ability to maintain water; causing drying/cracking of the skin which leads to scratching, this contributes proinflammatory mediators/prostaglandin release. C. Individuals with atopic dermatitis exhibit elevated serum IgE levels D. All of the above are associated with this disease

D. All of the above are associated with this disease

A 28 year presents today complaining of nausea, fatigue for 2 weeks. She states that she has being vomiting almost every morning for the last two weeks, normally in the morning and she feels tired throughout the day. She can't remember the last day of her menstrual cycle, but admits to being sexually active and using contraceptive intermittently. Urine beta-hCG is 1200 mIU/ml. All would be expect to be found on physical exam except? A. Chadwick sign is a bluish discoloration of the cervix B. Softening and enlargement of the cervix C. Breast changes D. All of the above would be expected findings on physical examination

D. All of the above would be expected findings on physical examination

According to the American Psychiatric Association's Diagnostic and Statistical Manual of Mental Disorders, Fifth Edition (DSM-5), phobic disorders are no longer a distinct group of anxiety disorders; nonetheless, they may still constitute a useful conceptual category. Which of the following diagnoses may be thought of as belonging to this category? A. Social anxiety disorder B. Specific phobia C. Agoraphobia D. All the above

D. All the above

65 year old male with a 35 pack-year smoking history presents to the office with dyspnea, chronic cough, sputum production, and a sensation of chest tightness for the past 2 months. On physical exam, you note an obese man with crackles at the lung bases, increased resonance on percussion, and an increase anteroposterior diameter of his chest. You give him a diagnosis of COPD and prescribe bronchodilators. What is the pathology of his disease? A. Reversible bronchoconstriction due to external triggers B. Inflammation of unknown etiology leading to multisystem noncaseating granulomas C. Fluid edema in lungs due to heart failure D. Alveolar wall destruction distal to to terminal bronchioles leading to permanent dilation E. Metastatic malignancy originating in the liver

D. Alveolar wall destruction distal to to terminal bronchioles leading to permanent dilation

All of the following are associated with Anemia of Chronic Disease EXCEPT: A. Significan't anemia in patients with chronic disease usually indicates coexisting iron or folate deficiency. B. Anemia associated with chronic disease usually is mild and remits with treatment of the disease. C. Clinical features are consistent with the underlying disease. D. Anemia is a hypochromic microcytic. E. All of associated with Anemia of Chronic Disease

D. Anemia is a hypochromic microcytic.

What is a preferred treatment for an incompetent cervix? A. Progesterone supplementation B. Cervical Cerclage C. Estrogen Supplementation D. Both A and B E. Both A and C

D. Both A and B

A 79-year-old man complains of gradual vision loss. He describes his vision as if he is looking through a "dirty window." On physical exam, there is no red reflex. What is the most likely diagnosis? A. Closed Angle Glaucoma B. Astigmatism C. Hyphema D. Cataracts

D. Cataracts

A 70 year old male complains of waking up abruptly in the night due to shortness of breath that is alleviated by sitting up. The patient admits to morning headaches, insomnia, and snoring. What diagnosis would you suspect? A. Asthma B. COPD C. Heart Failure D. Central Sleep Apnea

D. Central Sleep Apnea

A 26 year old female presents complaining of severe pain in her right eye and photosensitivity. On physical exam, you note inflammation of the conjunctiva, purulent exudate, and a white, oval area with sharply demarcated borders on the cornea. The patient wears contact lenses and admits to infrequent changings. What condition is most likely contributing to this patient's symptoms? A. Macular Degeneration B. Closed Angle Glaucoma C. Open Angle Glaucoma D. Corneal Ulcer E. Detached Retina

D. Corneal Ulcer

23 year old pregnant female comes in for her routine scheduled appointment with her OBGYN. With examination the doctor notices there is fetal tachycardia as well as repetitive variable decelerations. Upon further examination meconium is found in the amniotic fluid and there is fetal acidosis. What is the doctor suspicious of? A. Placenta abruption B. Cocaine use by the mother C. Miscarriage D. Fetal distress E. All of the above

D. Fetal distress

All of the following contribute to the sequence of a migraine, EXCEPT... A. Postdromal phase B. Headache C. Premonitary symptoms D. Fever E. Aura

D. Fever

A 23 year old female presents to the office today complaining of a new appearance of red bumps in the area of her labia. Upon taking a PMH you learn that she has recently become single and has been going out and partaking in sexual activity with multiple sex partners. She does admit to having itching and pain in the region approximately 10 days ago with some mild dysuria. She also admits to feeling flu-like symptoms and having mild muscle aches. Upon examination you observe 7-8 red papules on her labia majora with 3 painful ulcerative lesions. Her temperature is 100.4°F, BP 110/80, RR 16 bpm, HR 60 bpm. Gynecologic PMH is unremarkable. She has no previous hx of sexually transmitted infections or gynecological complications. LMP 3 weeks ago. No previous pregnancies. Upon waiting for the culture result you begin explaining to the patient that their most likely diagnosis is.... A. Syphilis B. Chlamydia C. Gonorrhea D. Genital Herpes

D. Genital Herpes

Chronic venous insufficiency is most commonly due to all of the following except: A. Malfunctioning (incompetent) valves in the veins. B. Past blood clot in the legs. C. Hypertension D. Hemolytic Anemia

D. Hemolytic Anemia

All of the following may be treatment used in Pylomyositis (MS) except... A. Physical therapy to improve muscle strength and function B. Corticosteroids and corticosteroid-sparing agents C. Rituximab D. High dose muscle relaxants and epidural injections

D. High dose muscle relaxants and epidural injections

All of the following are causes of pericardial effusion except: A. uremia B. malignancy C. AIDS D. IV catheters E. hypothyroidism

D. IV catheters

The mechanism likely involved with Lichen Planus is: A. Idiopathic B. Inflammatory C. Bacterial infection D. Immune-mediated E. Fungal infection

D. Immune-mediated

A patient comes in complaining that last week she had a URI and now it seems that she has an ear infection. The patient complains of ear pain, vertigo, dizziness, nausea, and tinnitus. Upon examination the patient has hearing loss and a fever. What should your number one differential diagnosis be? A. Menieres 'Syndrome B. Ototoxicity C. Otitis Media D. Labrynthitis

D. Labrynthitis

A rare, benign, smooth muscle neoplasm with an unclear pathology, but theorized to be caused by an autosomal dominant trait is known as: A. Malignant neoplasm B. Cyst C. Papule D. Leiomyoma E. carbuncle

D. Leiomyoma

Where is the most common location for pulmonary emboli to originate? A. Pelvic area B. Renal area C. Upper extremity D. Lower extremity

D. Lower extremity

What race is most affected by hyperglycemia? A. African Americans B. Caucasians C. Native Americans D. Mexican Americans E. Asians

D. Mexican Americans

Which of the follow age groups is pyloric stenosis typically seen in? A. >80 years old B. 50-70 years old C. 1-5 years old D. Newborn-6 months old E. 5-10 years old

D. Newborn-6 months old

Justin, a 17-year-old football player for his high school football team presents to the clinic today for a follow-up exam due to a head injury at Friday night's football game. He was hit forcefully after catching a football in the end zone and then was taken out of the game. On today's exam, the head CT was negative, and Justin is still C/O symptoms of fatigue, dizziness, frequent headaches rating a 6 on the pain scale, sensitivity to loud noise and a mild loss of memory. All vital signs are WNL. Your first differential diagnosis would be... A. Subarachnoid hemorrhage B. Epidural bleed C. Psychological symptoms D. Post-concussion syndrome

D. Post-concussion syndrome

When nystagmus develops in early childhood (not the later-in-life acquired form), what is a common pathophysiology? A. Stroke B. Multiple sclerosis C. Blow to the head and resulting hemorrhage D. Problem with the visual pathway from the eye to the brain

D. Problem with the visual pathway from the eye to the brain

A 50 year old male presents to the clinic for an annual physical and lab work. The patient states that he has no new complaints, except for some recent right eye irritation that he believes is more of a nuisance than a medical issue. Vital signs are normal. Upon physical exam, you notice a wedge-shaped growth extending onto his cornea, which begins at the outer corner of his eye. It appears to be soft, flat, and slightly white. You instruct him that he should have it surgically removed and refer him to ophthalmology. What is the diagnosis? A. Glaucoma B. Detached retina C. Retinoblastoma D. Pterygium E. Conjunctivitis

D. Pterygium

There are many causes of primary mitral regurgitation. In the United States only 2% of the population will suffer from this condition. Worldwide the leading cause of mitral regurgitation is: A. Trauma B. Congenital C. Lyme disease D. Rheumatic fever

D. Rheumatic fever

All of the following are symptoms of multiple sclerosis except: A. Diplopia B. Slurred speech C. Urinary urgency and frequency D. Rhinitis E. Dizziness and fatigue

D. Rhinitis

A patient presents to your office with complaints that their eyes and mouth are "really" dry. You note on the physical exam that the patient has conjunctivitis along with severe dry/scaly lips with cracks at the corners of the mouth. The patient states that this has happened throughout their lifetime with no known cause as of yet. The patient states that they do not have seasonal allergies, are not taking any anticholerginic drugs, and that their grandfather seemed to have the same issues. Given this history and exam findings, what is the MOST LIKELY cause of the patient's problems? A. Multiple Sclerosis B. Amyloidosis C. Atropine use D. Sjogren Syndrome

D. Sjogren Syndrome

Which of the following foods put you at risk for causing gastric cancer? A. Bananas B. Apples C. Kiwis D. Smoked Foods E. A, C and D

D. Smoked Foods

A 6 year old girl comes into the office with her mom presenting with a pruritic rash that began two days ago. Upon physical examination, you note macules, papules, pustules and vesicles that have crusted over present on the girl's face, scalp, and trunk. The lesions have a "dew drop on a rose petal appearance." What is the most likely diagnosis? A. Poison ivy B. Psoriasis C. Bullous pemphigae D. Varicella E. Roseola Infantum

D. Varicella

What can increase the risk of developing PAD(peripheral artery disease)? A. Smoking B. Diabetes C. High Blood Pressure D. all of the above

D. all of the above

All of the following are specific causes of pericarditis except: A. Idiopathic B. Infectious conditions C. inflammatory disorders D. all of the following are causes of pericarditis

D. all of the following are causes of pericarditis

How can you advice a patient to aid in avoiding Barotrauma to their eardrum? A. there is no advice to give B. Advice them to swallow C. Advice them to yawn D. both B and C are correct

D. both B and C are correct

Older people are at a higher risk of hip fracture due to all of the following except: A. osteoporosis B. balance problems C. poor vision D. increased vitamin D

D. increased vitamin D

A 10 year old male presents for a routine wellness exam. On physical exam you note the man has a short stature (Ht 4'10), large head with prominent forehead, a protruding jaw, bowed legs, and a forward curvature of the lower spine. Which hormonal condition can cause this patient's condition? a. Deficit of growth hormone c. Excess PTH b. Increase in growth hormone d. decreased ACTH

Deficit of growth hormone

A 70 year old African American man with a 15 year history of HIV presents to the emergency department in congestive heart failure. Which class of medication should be given to the patient immediately? a. Diuretics d. Nitrates b. Digoxin e. ARBs c. Hydralazine

Diuretics

A 19 year old male dishwasher came to the Urgent Care center presenting with "bumps" on his hands and on the sides of his fingers. He states that the bumps are extremely itchy. On exam, a cluster of fluid filled blisters that have a tapioca like appearance were noted. Based on the patient's history and findings on the exam, the patient most likely has which condition? a. Dyshidrosis d. Acne vulgaris b. Psoriasis e. Molluscum contagiousum c. Drug eruption

Dyshidrosis

Which of the following is most likely to cause Mastoiditis? A. S. pyogenes Group A, B. H. influenza C. S. pneumonia D. E. Coli E. A, B and C

E. A, B and C S. pyogenes Grp A H. influenza S. pneumonia

Otitis media is mainly due to Eustachian tube dysfunction caused by interference with the mucosa at the pharyngeal end of the Eustachian tube by edema, tumor, or negative intratympanic pressure that extends infectious processes from the nasopharynx into the middle ear. A common pathogen (s) that causes OM is: A. Streptococus pneumonia B. Haemophilus influenza C. M. catarrhalis D. A and B E. A, B, and C

E. A, B, and C

All of the flowing are possible finding on a physicial exam of a patient with Hydronephrosis, EXCEPT: A. Costoverterbral Angle Tenderness B. Edema C. Hypertension D. Hematuria E. All are possible physicial exam findings of a patient with hydronephrosis.

E. All are possible physicial exam findings of a patient with hydronephrosis.

All of the following are true of Ventricular Septal Defect EXCEPT: A. Congenital or acquired heart defect that involves the interventricular septum where there is communication of blood between the left and right ventricle B. VSD is the most common congenital heart malformation reported in infants and children. C. VSD can also occur as a complication of acute myocardial infarction (MI) D. Blood flow is typically left to right and is dependent on pulmonary vascular resistance and size of the defect E. All are true of Ventricular Septal Defect

E. All are true of Ventricular Septal Defect

All of the following are true regarding Crohn's Disease EXCEPT: A. Crohn's disease may involve both the small and large bowels as well as the mouth, esophagus, and stomach. B. Most commonly, the terminal ileum and right colon are involved, but the rectum frequently is spared. C. Skip lesions are characteristic. D. Abdominal cramps and diarrhea in a patient younger than 40 years are the most common presenting complaints. E. All are true regarding Crohn's Disease

E. All are true regarding Crohn's Disease

36 year old male patient made an appointment complaining of constipation and painful bowel movements. He has been noticing small specs of bright red blood on the toilet paper, which is concerning to him. Upon inspection of the anus you notice a tear in the mucosal lining that is red and irritated. What would you recommend to treat your patient? A. Increase fluid intake B. High fiber diet C. Sitz bath after BM D. Stool softeners E. All of the above

E. All of the above

A 3 year old African American male presents to your office for follow up of his second urinary tract infection in the last 6 months. The parents state he has been crying a lot more lately and holds his abdomen. Upon examination you note an abdominal mass and pain associated upon palpation. You discover he has stage III Wilms tumor, a rare kidney cancer. How will you treat this condition? a. Chemotherapy c. Radiation b. Nephrectomy d. All of the above

E. All of the above

Cervicitis can result from: A. gonorrhea B. chlamydia C. trichomoniasis D. allergic reactions to feminine hygiene products E. All of the above

E. All of the above

Retinal Artery Occlusion frequently occurs in individuals with a PMH of the which of the following? A. Diabetes B. Atrial fibrillation C. Heart valve disorders D. Hypertension E. All of the above

E. All of the above

A factor that may pathophysiologically contribute to developing developmental dysplasia of the hip include: A. Racial background B. Genetic predisposition C. Intrauterine positioning and sex D. Musculoskeletal disorders of intrauterine positioning E. All of the above are factors that contribute to DDH

E. All of the above are factors that contribute to DDH

All are forms of treatment for vulvar cancer except: A. Excision B. Partial vulvectomy C. Radical vulvectomy D. Chemotherapy and radiation E. All of the above are forms of treatment

E. All of the above are forms of treatment

Which of these conditions and etiologies are mismatched? A. Hiatal hernia: part of your stomach pushes upward through your diaphragm B. Esophageal varices: normal blood flow to the liver is obstructed by scar tissue in the liver or a clot. C. Esophageal spasms: painful muscle contractions in your esophagus prevent food and liquids from traveling to your stomach D. Gastroesophageal reflux disease: stomach acid or, occasionally, stomach content, flows back into your esophagus E. All of the above are matched correctly

E. All of the above are matched correctly

All of these increase your risk of having a syncopial episode except: A. Excessive alcohol intake B. Hypertension C. Advanced age D. Diabetes E. All of the above increase your risk

E. All of the above increase your risk

Which of the following conditions will contribute to the condition of tricuspid regurge? A. Right ventricle dilation due to heart failure, pulmonary hypertension or cardiomyopathy. B. Enstein's anomaly C. Infective endocarditis D. Carcinoid syndrome E. All of the above will contribute to tricuspid regurge

E. All of the above will contribute to tricuspid regurge

65 year old female presents with nausea, vomiting, and a headache that is exacerbated whenever she coughs x 3 days. She has been having diplopia for several months now as well. Her visual acuity is intact. Upon fundoscopic examination you notice cotton wool spots, peripapillary hemorrhages, disc margins are obscured, and the disc is grossly elevated. The patient has papilledema. Which of the following would NOT be in your differential of the cause? A. Brain tumor B. Idiopathic intracranial hypertension C. Decreased CSF resorption D. Encephalitis E. All of the above would be in the differential

E. All of the above would be in the differential

In premature infants, respiratory distress syndrome develops because of impaired surfactant synthesis and secretion leading to all of the following except: A. atelectasis B. ventilation-perfusion (V/Q) inequality C. hypoxemia D. hypoventilation E. All of the following are complications of RDS

E. All of the following are complications of RDS

Which of the following are risk factors for placenta previa? A. Advanced maternal age(>35 y/o) B. Smoking C. Previous cesarean delivery D. Nonwhite ethniticy E. All the above

E. All the above

24 year old male presents to the office with complaints of low back pain for the past 1 year. He states that it started as a mild pain and it has slowly progressed to moderate-severe over the past year. Now, he states that he has problem moving his shoulders and hips when he goes to the gym. Physical exam reveals limited spinal range of motion of the cervical and thoracic spine segments. Chest and lower back X-Ray shows fusion of the spine, with an apparent "bamboo spine" appearance. What is the diagnosis? A. Fibromyalgia B. Vertebral compression C. Cauda Equine Syndrome D. Rheumatoid Arthritis E. Ankylosing Spondylitis

E. Ankylosing Spondylitis

Which of the following is not a sign or symptom of Blepharitis? A. Burning, watering or red eyes and eye lids B. Photophobia C. Pain or crusting of the lashes and medial canthus D. A feeling of having a foreign body in the eye E. Conjunctival hemorrhage

E. Conjunctival hemorrhage

A 25 year old female presents with a "painful bump" on her left eyelid. She denies any radiation of the pain. On physical exam of around the orbit, the eye, and conjunctival surface you find a localized tender area of swelling with a pointing eruption on the external eyelid. No laboratory studies are indicated. What is your diagnosis? a. Hordeolum c. Corneal abrasion b. Conjunctivitis d. chalazion

Hordeolum

64 year old male with PMH of HTN, Type 2 DM, and Hyperlipidemia presents to the ED with sharp chest pain which started 6 hours ago when he was shoveling snow in his driveway. He describes the pain located on the left side of his chest, which radiates down his left arm, and feels like "an elephant is sitting on his chest." On physical exam, the patient presents with tachycardia and cool, clammy, pale skin. You are concerned that his symptoms could be due to a possible acute Myocardial Infarction. What would be the first test to order to help confirm your suspicions? A. Chest X-Ray B. Spirometry C. Percutaneous Coronary Intervention D. CT Chest E. EKG

E. EKG

45 year old female presents with difficulty of swallowing foods and speaking for the past 1 month. She also reports unintentional weight loss, but reports that she is still eating meals adequately, albeit with difficulty swallowing. Physical examination reveals an unusually firm mass in the anterior neck with irregular contours. The mass is fixed and does not move upon swallowing. What is the next best step in managing this neck mass? A. Total thyroidectomy B. Thyroid lobectomy C. Chemotherapy D. Radiation E. Fine needle aspiration with cytology studies

E. Fine needle aspiration with cytology studies

67 year old female with PMH of Type II DM presents to your office for her annual check up. She admits to being noncompliant on her insulin treatments, but tries to eat healthier. Despite her efforts, her HbA1c level is 8.0%. What symptom(s) would she be most prone to suffering from? A. Diarrhea B. Constipation C. Dry cough D. Epistaxis E. Peripheral Neuropathy

E. Peripheral Neuropathy

An 18 year old African American female presents to your office with a history of GERD x2 years and a current sore throat that has lasted roughly 2 weeks. It has not responded to OTC medication or herbal remedies. She is also noticing chest pain when eating. You suspect: A. Hiatal hernia B. Vitamin B deficiency C. A new allergy D. Mitral valve stenosis E. esophagitis

E. esophagitis

A 40 year old male complains of "shaky hands." He states that he noticed the trembling two months ago and it has gotten worse. He states that he has difficulty writing, shaving, tying his shoelaces or drinking a glass of water because of the tremors. He states the tremors are only worsened with movement and denies involvement of his legs. What is your most likely diagnosis? a. Essential tremor c. Dystonia b. Epilepsy d. Parkinson's disease

Essential Tremor

True or false: Renal cell carcinoma is the least common type of kidney cancer in adults.

False

Vitiligo is a depigmentation of skin but strictly affects arms, legs, chest and back. True/False

False

A 45 year old female presents with abdominal pain, gas and diarrhea for the past several days. She states that she has had fecal urgency and incomplete bowel movements. Eating seems to make her symptoms worse and antidiarrheal medication has helped relieve her symptoms. She also admits that this has happened several times in the past and it normally subsides without treatment. She denies rectal bleeding or weight loss. What is the most likely diagnosis? a. Irritable bowel syndrome c. Colorectal cancer b. Crohn's disease d. Ulcerative colitis

Irritable bowel syndrome

S.K. age 5 presents with generalized edema with a puffy face, distended abdomen, and edematous legs. He has gained weight but has eaten very little during the past week and has been quite irritable and lethargic. His blood pressure is normal. Laboratory tests indicate high levels of albumin, lipids, and protein casts in the urine, which has a high specific gravity. Blood tests show hypoalbuminemia and elevated cholesterol levels. Which condition would you suspect? a. Nephrotic syndrome d. Diabetes insipidus b. Cushing's syndrome e. Hyperthyroidism c. Hypothyroidism

Nephrotic syndrome

A 30 year old male presents with abdominal pain, stomach cramps, constipation, nausea, vomiting and rectal bleeding for the past 2 weeks. He also admits to a "warm" feeling in his face and neck. What should you do next? a. Order appropriate labs/tests to check for a Carcinoid tumor b. Send patient home with laxatives to help with the constipation. c. Order appropriate labs/tests to check for pheochromocytoma d. Order appropriate labs/tests to check for adenocarcinoma e. All of the above

Order appropriate labs/tests to check for a Carcinoid tumor

Is the following statement T/F regarding pertussis Pathophysiology? The bacterium B. pertussis attaches to the cilia of the respiratory epithelial cells, produce toxins that paralyze the cilia, and cause inflammation of the respiratory tract, which interferes with the clearing of pulmonary secretions.

True

True or false: Disseminated intravascular coagulation is always secondary to an underlying disorder.

True

What is the most likely description of a patient who could develop lupus? a. 25 year old (childbearing age), African American woman b. 68 year old, White woman c. 29 year old, Asian male d. 68 year old, African American male

a. 25 year old (childbearing age), African American woman

Which of the following statements is true about restrictive cardio: a. A condition in which the walls of the ventricles become abnormally rigid and lack the flexibility necessary to expand while they fill. b. Prognosis is usually worse in adults rather than in children c. Treatment doesn't usually include heart transplant d. All of the above

a. A condition in which the walls of the ventricles become abnormally rigid and lack the flexibility necessary to expand while they fill.

A 40-year old Hispanic woman comes in to the clinic complaining of darkened, itchy areas in the neck and armpits that have developed over the last 3-4 months. In the patient's history you are made aware that the patient is diabetic and although she is under medications, does not have a very good control of her condition. On the physical exam you encounter an obese woman, with symmetric, thickened, hyperpigmented, velvety plaques in the back of her neck and in her armpits. The most likely diagnosis for this patient's condition is? a. Acanthosis nigricans d. Seborrheic Keratosis b. Contact dermatitis e. Ichthyosis vulgaris c. Melanoma

a. Acanthosis nigricans

An excess of Growth Hormone (GH) in adulthood would be considered which of the following disorders? a. Acromegaly c. Achondroplasia b. Gigantism d. Myxedema

a. Acromegaly

Patient presents to the ED with delayed responses and lethargy. Patient is not responding well to questions and can barely keep her eyes open. Upon getting the history from her family, it was stated that patient has a history of depression and family thinks she might have overdosed on her medication. Overdosing on her medication led to what? a. Altered level of consciousness c. Dementia b. Fatigue d. Respiratory distress

a. Altered level of consciousness

Mr. Cool comes into your office complaining that he has had intermittent chest pain the past couple of days. Patient is a healthy 35 year old Caucasian male with no history of CAD in his family. He mentions he started a new job 2 weeks ago that has caused a lot of stress. Which of the following terms refers to the chest pain brought on by physical or emotional stress and relieved by rest or medication? a. Angina pectoris c. Atheroma b. Atherosclerosis d. Ischemia

a. Angina pectoris

Which of the following diagnosis below fit the following description: involves a young person (teenager or young adult) who is mildly overweight or of normal weight and who begins a diet and exercise plan to lose weight. As he or she loses weight and receives initial positive reinforcement for this behavior (eg, compliments by peers on his or her appearance), the reward is high and causes an inability to stop this behavior once an ideal weight is achieved. a. Anorexia Nervosa c. Depression b. HIV d. Bipolar disorder

a. Anorexia Nervosa

The patient has the following: Painless gross hematuria - Approximately 80-90% of patients; classic presentation, Irritative bladder symptoms (eg, dysuria, urgency, frequency of urination) - 20-30% of patients, Pelvic or bony pain, lower-extremity edema, or flank pain? a. Bladder cancer c. UTI b. Ovarian cancer d. HIV

a. Bladder cancer

The most common cause of TIA includes: a. Blood clot c. Lack of O2 in the brain b. Sudden decrease in blood pressure d. Bacterial infection (meningitis)

a. Blood clot

10 y/o patient presents to the ED with symptoms of malaise, fever and pain on the side of his face. Upon examination of the patient, you notice swelling and erythema on the overlying skin of the face. You also notice small yellow curds of saliva discharge. What is the most probable diagnosis? a. Chronic punctate parotitis c. Allergic reaction b. HIV parotitis d. Rubella

a. Chronic punctate parotitis

Which one of the following is a risk factor for developing testicular cancer? a. Cryptorchidism d. Paraphimosis b. Varicocele e. Hypospadias c. Hydrocele

a. Cryptorchidism

Which of the following is true about the treatment of hyphema? a. Customary treatment includes hospitalization, topical steroids, and systemic steroids b. Always treat with antibiotics c. An eye patch and shield is usually not recommended d. Hyphema does not require treatment

a. Customary treatment includes hospitalization, topical steroids, and systemic steroids

Which of the following is the most appropriate surgical method to perform an abortion in the second trimester? a. Dilation and evacuation b. Manual vacuum aspiration c. Induced abortion d. Abortion is not an option in the second trimester

a. Dilation and evacuation

Fever, Inguinal or axillary lymphadenopathy, Testicular and/or inguinal pain, Skin exfoliation, Limb or genital swelling are signs and symptoms of which of the following? a. Elephantiasis c. Lyme disease b. TB d. Aplastic anemia

a. Elephantiasis

The most common type of fracture in the forearm is? a. Fall on an outstretched arm c. Fighting b. Trauma d. None of the above

a. Fall on an outstretched arm

Cardiac arrhythmias, including tachycardias and bradycardias, can be observed as a result of autonomic nervous system involvement. Tachypnea may be a sign of ongoing dyspnea and progressive respiratory failure. Blood pressure lability is another common feature, with alterations between hypertension and hypotension. Temperature may be elevated or low. Respiratory examination may be remarkable for poor inspiratory effort or diminished breath sounds. On abdominal examination, paucity or absence of bowel sounds suggests paralytic ileus. Suprapubic tenderness or fullness may be suggestive of urinary retention. What is this describing? a. Guillain-Barre c. UTI b. Anaphylaxis shock d. PID

a. Guillain-Barre

A 30-year-old female presents with a "nodule" on her eyelid. She reports that it started as a small lesion and has been progressively increasing in size over several weeks. It is mildly painful, erythema and swelling is present. What is the most appropriate next step in the management of this lesion? a. Instruct the patient to use a hot compress on her eye for several days b. Start a 1-week course of topical antibiotics c. Biopsy to rule out malignancy d. Excise the lesion e. Observation

a. Instruct the patient to use a hot compress on her eye for several days

A patient presents to your cardiovascular office after being referred from him primary care physician for a heart murmur, Shortness of breath, especially during exertion, Chest pain, syncope, and fatigue. You diagnose him with pulmonary stenosis. What is the most likely cause? a. Isolated valvular c. Peripheral obstruction b. Subvalvular d. All of the above

a. Isolated valvular

Which of the following is true about essential hypertension? a. It tends to develop gradually over a number of years b. It is caused by an underlying condition c. Caucasians are more likely to have essential hypertension d. Hypertension can not be well-managed

a. It tends to develop gradually over a number of years

A fall where a patient landed strait on their wrist would most likely indicate an injury to which soft tissue? a. Ligament c. Skin b. Muscle d. Blood vessel

a. Ligament

A 7-year-old boy comes into your practice. His mother says he has been itching like crazy. Upon physical exam you notice raised, round, flesh-colored lesions with small umbilications on his face, neck, and the tops of his hands. What is the most-likely diagnosis? a. Mollsucum contagiosum c. Herpes simplex b. Varicella d. Measles

a. Mollsucum contagiosum

The following symptoms: difficulty sleeping, tense feelings, irritability, clumsiness, mood swings, craving and anxiety indicate which of the diagnosis below: a. PMS c. BPH b. PID d. UTI

a. PMS

A solitary "coin lesion" appears on the chest xray of a 55 y/o male. What is the most likely diagnosis? a. Pulmonary nodule c. Pneumonia b. Foreign object aspiration d. Bronchogenic adenocarcinoma

a. Pulmonary nodule

A patient presents to the ED with joint pain in the knees, ankles and feet. Patient is also complaining of an irritated eye, which was diagnosed as conjunctivitis. Patient states prior to the pain she experienced a urinary tract infection. What is the most probable diagnosis? a. Reactive arthritis c. Kidney infection b. Herpes Simplex Virus d. Chlamydia

a. Reactive arthritis

What disease is most likely to predispose someone to tricuspid stenosis? a. Rheumatic Fever c. Lyme Disease b. Hepatitis C d. MRSA

a. Rheumatic Fever

In chronic Cor pulmonale which of these pathologies generally predominate? a. Right Ventricular Hypertrophy c. Left Ventricular Hypertrophy b. Pulmonary Vasoconstriction d. ARDS

a. Right Ventricular Hypertrophy

A man and woman in their 20s have unsuccessfully been trying to conceive for the last year. The woman has regular menses and a 28-day cycle. In the initial evaluation, which of the following tests or evaluations should be considered first-line? a. Semen analysis c. Hysterosalpingogram b. Postcoital testing d. Endometrial biopsy

a. Semen analysis

Patients with an infected joint typically present with the triad of fever (40-60% of cases), pain (75% of cases), and impaired range of motion. These symptoms may evolve over a few days to a few weeks. Fever is usually low-grade (< 102°F), with rigors present in only 20% of cases. What does the patient have? a. Septic arthritis c. None of the above b. Rock mountain fever d. All of the above

a. Septic arthritis

A patient presents with a skin lesion that he describes as painless when he first noticed it, but now, about 3 hours later, describes it as quite painful. Physical exam reveals a ring of pallor immediately around the lesion with a surrounding area of erythema. The patient reports recently camping out doors in the warm, humid Florida everglades prior to coming in. What is your likely diagnosis? a. Spider Bite c. Allergic dermatitis b. Erythema nodosum d. Pemphigus Vulgaris

a. Spider Bite

A 28 year old male baseball player has been pitching for the past 10 years. He comes into the office with a cc of pain in his shoulder. He states he noticed the pain one month ago after a baseball game and it has progressively gotten worse. He states that the pain is made worse when he lifts his arm and Advil has helped to alleviate the pain. On physical exam you note the shoulder is warm, red and inflamed. You inform the patient that participating in any type of sports or physical activity can result in: a. Sprains b. Strains c. Tendonitis d. Bursitis e. All are examples of soft tissue injuries that can result from athletic activities.

a. Sprains b. Strains c. Tendonitis d. Bursitis *e. All are examples of soft tissue injuries that can result from athletic activities. *

A 56 year old, male, fair skin patient who gardens frequently comes into the dermatology office for a routine checkup. Upon examination the practitioner notices a firm, red, flat sore with scaly crust. The lesion was located on the patient's left ear. The lesion does not cause any pain for the patient. What is the most likely diagnosis? a. Squamous cell carcinoma c. Actinic Keratosis b. Basal cell d. Seborrheic Keratosis

a. Squamous cell carcinoma

The main pathogen that causes tooth decay in the human mouth is: a. Streptococcus mutans c. Strep. pyogenes b. Staph. Aureus d. E. coli

a. Streptococcus mutans

Which of the following is the most common etiologic agent of bacterial meningitis in the pediatric population of the US? a. Streptococcus pneumonia c. Listeria monocytogenes b. Haemophilus influenza Type B d. Neisseria meningitides

a. Streptococcus pneumonia

Which dermatologic disorder is characterized by widespread erythema, necrosis, and bullous detachment of the epidermis and mucous membranes, resulting in exfoliation and possible sepsis and/or death? a. Toxic epidermal necrosis c. Pemphigous vulgaris b. Psoriasis d. Vitiligo

a. Toxic epidermal necrosis

Which of the following urinary findings is suggestive of acute glomerulonephritis? a. red cells and red cell casts d. oval fat bodies b. white cells and white cell casts e. hyaline casts c. renal tubular epithelial cells

a. red cells and red cell casts

Thrombotic Thrombocytopenic Purpura is a rare blood disorder that consists of clotting in small blood vessels and a low platelet count. It can be inherited or acquired and is caused by a lack of a certain enzyme activity. That enzyme is: a. BRACA1 d. Fibrin b. ADAMST13 e. None of the above c. Protease

b. ADAMST13

A 64 year old Caucasian male comes into your office in Ft. Lauderdale for a routine check-up. He has dark skin and blue eyes. His HR was 68 bpm, BP was 190/82, RR was 14, and his temperature was 98 F. You notice a dry, scaly, pink, slightly raised patch on the auricle of his ear lobe. He claims it has been there for several years and he hasn't noticed any sudden changes lately. He does not complain of any pain, but occasionally it itches. You suspect it is most likely: a. Squamous cell carcinoma d. Macule b. Actinic keratosis e. Eczema c. Psoriasis

b. Actinic keratosis

A patient comes in to your office with generalized complaints of fatigue and lethargy. Upon physical exam, you notice that the skin is pale and unusually bruised. Upon questioning, the patient admits to shortness of breath and unusual bleeding from the gums along with excessive nose bleeds lately. A bone marrow aspirate shows the presence of Auer rods and a significant increase in myeloblasts, along with a CBC showing a low hematocrit count and thrombocytopenia. What is your most likely diagnoses of the patient? a. Sickle Cell Anemia c. Pernicious Anemia b. Acute Myelogenous Leukemia d. Chronic Lymphocytic Anemia

b. Acute Myelogenous Leukemia

Which ligament is most commonly injured in ankle sprains? a. Posterior talofibular ligament c. Calcaneofibular ligament b. Anterior talofibular ligament d. Deltoid ligament

b. Anterior talofibular ligament

Which of the following is NOT one of the four parts of Tetralogy of Fallot ? a. Ventricular septal defect c. Right ventricular outflow obstruction b. Atrial septal defect d. Overriding aorta

b. Atrial septal defect

Patient comes in complaining of back pain that has lasted a week. He mentions that last week he was playing golf with some of his friends and remembers feeling a sharp pain when taking his drive on the 18th hole. Pain is located along right border of his thoracic vertebrae with no radiating pain noted. His ROM is limited due to pain. Patient has no tenderness upon palpation of spinous processes, but point tenderness along paraspinal muscles. Vitals are all WNL. Which of the following is the most likely diagnosis? a. Spinal Stenosis c. Herniated T6-7 b. Back sprain/strain d. Vertebral fracture

b. Back sprain/strain

Which of the following tumor markers (biomarkers) is usually associated with ovarian cancer? a. CA19-9 d. Alpha-fetoprotein b. CA125 e. P53 c. PSA

b. CA125

Which one of the following is true concerning the characteristics of childhood autism? a. Kids with autism do well with changing routines. b. Childhood autism is usually diagnosed before the age of 3 years. c. Females are more affected than males. d. Environmental factors likely do not play a role in this condition. e. Benzodiazepines are a mainstay of treatment.

b. Childhood autism is usually diagnosed before the age of 3 years.

Upon performing a HEENT physical examination on a patient who's chief complaint was "I feel like I can't hear as well", you notice keratinized debris is evident above the tympanic membrane. By using just this information, what is your most likely diagnoses? a. Fungal otitis externa b. Cholesteatoma c. Eczematous otitis media d. Chronically sclerosed tympanic membrane

b. Cholesteatoma

Ms. Johnson comes into the health clinic complaining of excruciating pain in her right arm. After getting a brief history from her, she states that she had fractured that same arm last year and it had completely healed. Nothing comes to mind as to how the pain started. She presents with slight edema and discoloration in the affected area, muscular atrophy, and decreased ROM in her affected arm. She also mentions she has extreme sensitivity to pain in the affected area. Which if the following is the most likely mechanism for her symptoms? a. Phantom Limb c. Thrombosis b. Complex Regional Pain Syndrome d. Broken Humerus

b. Complex Regional Pain Syndrome

Patient has the following: Unilateral, severe pain, redness, and pressure in the supratemporal region of the orbit, Rapid onset (hours to days), Can be bilateral, painless enlargement of the lacrimal gland present for more than a month. What does the patient have? a. Mydriasis c. Conjunctivitis b. Dacryoadenitis d. None of the above

b. Dacryoadenitis

Patient presents to the clinic with symptoms of flat and scaly area on the skin which is red and itchy. The patch develops a slightly raised border and expands forward, forming a circular ring. Upon examination of the patient, the provider notices the lesions are irregular, resembling a snake or worm like pattern. What does the patient present with? a. Pityriasis Rosea c. Psoriasis b. Dermatophyte infection d. Lichen planus

b. Dermatophyte infection

Which of the following is not a main interplay factor that acne vulgaris develops from? a. Excess sebum production b. Excess sunlight exposure c. Follicular epidermal hyperproliferation d. Presence of commensal bacteria

b. Excess sunlight exposure

A 30-year-old male presents with symptoms of SOB, chest pain, and dizziness. Patient's physical exam shows findings of double apical impulse and a systolic ejection. Patient states he has a history of abnormal myocardial calcium kinetics. What is the most probably diagnosis? a. MI c. Murmur b. Hypertrophic Cardiomyopathy d. COPD

b. Hypertrophic Cardiomyopathy

A 65 year Caucasian female comes into the health clinic complaining of increased fatigue and weight gain over the past couple of weeks. She has no past medical history and reports no changes in her diet. Aside from slight enlargement in her neck area, she has no physical abnormalities. After a thorough H&P the physician orders a blood test, which shows elevated TSH levels and decreased free hormone levels. She is really concerned and wants to know what is going on. Which if the following is the most likely mechanism for her symptoms? a. Cancer c. Hyperthyroidism b. Hypothyroidism d. Diabetes

b. Hypothyroidism

Which of the following is true about diabetes insipidus? a. It is a very common disorder b. It is characterized by polydipsia c. It is related to diabetes mellitus d. One common symptom is the inability to urinate

b. It is characterized by polydipsia

Patient comes into the ER complaining of extreme malaise and fatigue. He reports that he has been throwing up all morning. After getting the patients history, you order an ABG and results show that the patients PH is 7.55. What is the result from the patient's condition? a. Metabolic Acidosis c. Respiratory Acidosis b. Metabolic Alkalosis d. Respiratory Alkalosis

b. Metabolic Alkalosis

Olivia a 28 year old female arrives at your office with complaint of pain in her abdomen. When questioned when the pain began she said before the end of her menstrual cycle. She also explained that she has dyspareunia, pain during intercourse. As a provider you are concerned and order an ultrasound and a urine pregnancy test. Pregnancy test is negative and she has not had intercourse due to pain in 3 weeks. On ultrasound a fluid filled mass was noted. What is the most likely diagnosis? a. Ectopic pregnancy c. Bladder Cancer b. Ovarian Cyst d. Herpes Simplex 2

b. Ovarian Cyst

A 64 year old man is complaining of bone pain, especially in his spine, nausea, loss of appetite, confusion, fatigue, frequent infections, weight loss and weakness in his legs. The doctor suspects bone cancer, multiple myeloma. What is the cell that is responsible for that cancer? a. Osteoblast c. Osteoblast b. Plasma Cell d. All of the above

b. Plasma Cell

Ms. Johnson comes into the health clinic complaining of shortness of breath, chest pain and anon-productive cough that started last night. She mentions that she fell down while carrying her groceries earlier that day and landed on her chest. Physical examination reveals dullness percussion in lower lobe of right lung as well as decreased tactile fremitus. Which if the following is the most likely mechanism for her symptoms? a. Pulmonary congestion c. Bronchospasm b. Pleural effusion d. Pneumonia

b. Pleural effusion

A 60-year-old male with a history of hypertension and hepatitis B presents to his primary care physician complaining of several months of fever, weakness, myalgias, and arthralgias. Home medications include lisinopril. Biopsy findings from the mesenteric vasculature. No evidence of involvement of microscopic vessels, pulmonary vessels, or glomerulonephritis is found. Which of the following is the most likely diagnosis? a. Scleroderma c. Rheumatoid Arthritis b. Polyarteritis nodosa (PAN) d. Gout

b. Polyarteritis nodosa (PAN)

Which of the following food types would you want to avoid or limit MOST if you had PKU? a. Fruits c. Carbohydrates b. Proteins d. Lipids

b. Proteins

Ms. Johnson comes into the health clinic complaining of weird changes in her right eye. Aside from diabetes, she is a healthy middle aged African American woman. She states that she hasn't hurt her eye doing anything. She complains of little "squiggly lines" in her line of sight along with flashes. She is really concerned and wants to know what is going on. Which if the following is the most likely mechanism for her symptoms? a. Conjunctivitis c. Myasthenia Gravis b. Retinal Detachment d. Myopia

b. Retinal Detachment

Which is false? a. Hepatitis C is an infection caused by the hepatitis C virus (HCV) that attacks the liver and leads to inflammation. b. The World Health Organization (WHO) estimates that about 20% of the world's population has been infected with HCV c. There are more than 170 million chronic carriers who are at risk of developing liver cirrhosis and/or liver cancer. d. All are true

b. The World Health Organization (WHO) estimates that about 20% of the world's population has been infected with HCV

Why are patients with diagnosed panic disorder usually noncompliant with their medication? a. The patient is not fond of the taste b. They fear losing control of themselves c. They prefer their own coping mechanisms d. They trust their primary care provider more then their psychiatrist

b. They fear losing control of themselves

All of the following are true about Galactorrhea diagnosis and treatment except: a. It's a milky nipple discharge unrelated to the normal milk production of breast-feeding b. This condition is only seen on women c. It results from overproduction of prolactin, a hormone secreted by the pituitary gland d. All of the above

b. This condition is only seen on women

A 50 y/o woman presents to her primary car provider with a rash that has raised, erythematous wheals. The rash is pruritic and blanches on palpation. The patient mentions that she recently started taking a diuretic for her HTN. What is the most likely diagnosis? a. Li Lichen Simplex Chronicus c. Lyme Disease b. Urticaria d. Atopic dermatitis

b. Urticaria

Patient comes in with the following signs and symptoms, Small, fleshy, grainy bumps; Flesh-colored, white, pink or tan; Rough to the touch; Sprinkled with black pinpoints, which are small, clotted blood vessels. What does the patient have? a. Cancer c. TB b. Verrucae d. All of the above

b. Verrucae

A patient presents to you clinic c/o dizziness when he turns his head. He states that when he sits down, the dizziness goes away rather quickly. When you are talking to the patient you notice nystagmus and that he cannot control his eye movements. Upon getting the patients medical history you find that he recently underwent surgery on his ear. What is the most likely diagnosis? a. Astrocytoma c. Hematoma b. Vertigo d. Viral infection of the ear

b. Vertigo

Patient has the following: "Allergic shiners" are dark circles around the eyes and are related to vasodilation or nasal congestion. "Nasal crease" is a horizontal crease across the lower half of the bridge of the nose that is caused by repeated upward rubbing of the tip of the nose by the palm of the hand. What does the patient have? a. Influenza c. Allergic Rhinitis b. Bronchitis d. Lung cancer

c. Allergic Rhinitis

A 45-year-old computer programmer presented to his family physician with a 3-year history of gradual-onset "ringing" in both ears. The tinnitus was constant and was more noticeable when he went to bed. All vital signs were within normal limits. Otoscopy revealed normal external auditory canals and clear tympanic membranes with normal mobility on pneumatic insufflation. His facial nerve function was grossly normal. A Weber tuning fork test lateralized to the left, and the Rinne tuning fork test was positive bilaterally. The remainder of the head and neck examination was unremarkable. Which of the following would be a reasonable first-line option for management in this case? a. Observation for another year to see whether the tinnitus improves spontaneously b. Noncontrast CT of the temporal bones c. Audiologic testing d. Tympanostomy tube placement

c. Audiologic testing

A 40-year-old woman brings in her 70-year-old father concerned that he is being abused at his nursing home because he has a lot of bruising and is losing weight. Upon entering the room you find an older male, pale, thin, and looking very fatigued. Upon examination you find splenomegaly and bruising. His CBC shows leukocytosis and upon further testing you find a Philadelphia chromosome on chromosome 22. What is the most likely diagnosis? a. Lymphoma b. AML c. CML d. Abuse from the staff at the nursing home

c. CML

A patient presents to you eye-clinic c/o severe eye pain, tears, and says he feels like something is in his eye when it's shut. When examining him you notice and the light from your otoscope bothers the affected eye. What is your most likely diagnosis? a. Foreign body obstruction c. Corneal abrasion b. Damage to CN V1 d. Detached retina

c. Corneal abrasion

Which one of the following conditions is not associated with the development of avascular necrosis (of hip)? a. Trauma d. Steroids b. Sickle Cell Disease e. Alcohol Use c. Emphysema

c. Emphysema

Which of the following is true about macrocytic anemias? a. Causes include poor absorption of vitamin B12 in the stomach due to prior gastrectomy b. Schilling test is used to diagnosis folate deficiency, c. Folate defiency can be caused by celiac disease or Crohn's disease d. Strict vegetarians are at risk of folate deficiency and may need chronic supplementation.

c. Folate defiency can be caused by celiac disease or Crohn's disease

A 35-year-old male patient presents to you office with complaints of N/V and fatigue and RUQ tenderness. He tells you that he returned from a trip overseas about 3 weeks ago. You don't suspect that you will have to treat this patient since it seems like a mild case. You do a blood test to confirm your diagnosis. What diagnosis are you suspecting? a. Mononucleosis c. Hepatitis A b. Giardia d. Influenza

c. Hepatitis A

A 68 year old male presents complaining of fever, fatigue and loss of appetite. While taking his history his wife tells you that he had to have some heart surgery about a year ago, but she doesn't know what they did. On physical exam you hear a slight murmur. You also notice non-tender, macular lesions on the palms of his hands and soles of his feet. Recalling your extensive PA school education, you recall that these are called Janeway lesions and are most often associated with: a. Syphilis d. Giant cell arteritis b. Erythema nodosum e. Rheumatic fever c. Infective Endocarditis

c. Infective Endocarditis

A 15 month-old baby boy is brought to the emergency department with severe colicky abdominal pain followed by vomiting. A tubular mass is palpated in the abdomen and rectal examination reveals bloody mucus. Which of the following is the most likely diagnosis? a. Appendicitis c. Intussusception b. Infectious enteritis d. Pyloric stenosis

c. Intussusception

One of the most common symptoms of Acute Lymphocytic Leukemia (ALL) is bone pain, which is due to the overcrowding, abnormal growth, and rapid division of what? a. Neutrophils c. Lymphoblasts b. RBC's d. Cytobones

c. Lymphoblasts

Patient presents to the ER post epileptic seizure. Now with vomiting, with much force, the mother explained. Labs drawn indicate a low reticulocyte count. On physical exam blood was noted in the throat and oral cavity. What is the most likely diagnosis? a. Irritable bowel syndrome c. Mallory-Weiss Tear b. Liver cancer d. stomach ulcer

c. Mallory-Weiss Tear

Which of the following best describes the clinical characteristics of abruption placenta? a. Variable amount of blood loss, no pain, and normal fetal heart rate, with no significant maternal history. b. Scant blood loss, soft and nontender uterus, grand multiparity c. Moderate amount of blood loss, uterine hypertonus, history of maternal HTN d. Bloody mucous plug, regular contractions

c. Moderate amount of blood loss, uterine hypertonus, history of maternal HTN

Patient presents to ER with SOB and a cough. History obtained indicates he has worked in the coal industry. What is the most likely diagnosis? a. Bronchitis c. Pneumoconiosis b. Strep throat d. viral infection of the lungs

c. Pneumoconiosis

Which of the following suggests Angina Pectoris? a. Internal hemorrhage c. Positive Levine sign b. Pulmonary infiltrates d. Localized pain in the fingers

c. Positive Levine sign

A 47 year old Hispanic female presents with a 12 pack year history and COPD. Over the past week she has felt persistant fatigue, bouts of dizziness, and has noticed considerable swelling in her ankles. Before taking her vitals, you highly suspect she has the following condition: a. Acute arterial stenosis d. Rheumatic fever b. Congestive heart failure e. Influenza c. Pulmonary hypertension

c. Pulmonary hypertension

A 32y/o woman comes into the ER saying that suddenly she felt something wet on her legs and noticed she "peed her pants." Upon taking a history she informs you that she was relaxing with her family when the incident took place. She also tells you she has been receiving pain injections in her back for extreme lower back pain. Her last injection was last week. Her pain level is 8/10 and it is radiating down her right leg. After receiving imaging study results, you realize this is not a small problem. Who would you consult? a. Urology c. Surgery b. Pain Management d. None of the above

c. Surgery

A 60 year old retired man just returned from a flight across the world in China where he was vacationing. He presents to the clinic after his long, 12 hour flight with a CC of a warm, tender, painful area on his right calf. He admits to there being pain upon palpation and rates it a 6 out of 10. He also admits that he accidentally struck that same area on the sofa this morning before visiting the clinic. Upon further history taking, the man admits to having a pacemaker as well. Before obtaining any other information or running any labs or tests, what is your most likely differential diagnoses? a. Fractured Fibula c. Thrombophlebitis b. Torn Achilles Tendon d. Dermatitis

c. Thrombophlebitis

Thickening of the epidermis secondary to scratching best describes: a. lichen striatus c. lichen simplex chronicus b. lichen planus d. lichen nitidus

c. lichen simplex chronicus

A 78 year old male was admitted to the hospital two days ago with pneumonia. He is now in the ICU and showing signs of tachycardia, tachypnea. He requires increasingly high concentrations of inspired oxygen. What is the most likely diagnosis? a. Sepsis b. Pulmonary hypertension c. Advanced stage pneumonia d. Acute respiratory distress syndrome e. Myocardial infarction

d. Acute respiratory distress syndrome

Which is false? a. In children, cardiac tamponade is more common in boys than in girls, with a male-to-female ratio of 7:3. b. In adults, cardiac tamponade appears to be slightly more common in men than in women. c. Cardiac tamponade related to trauma or HIV is more common in young adults, whereas tamponade due to malignancy and/or renal failure occurs more frequently in elderly individuals d. All are true

d. All are true

All of the following are true about acute pancreatitis diagnosis and treatment except: a. The two most common causes of pancreatitis in the US are: heavy alcohol use and gallstones. b. The main symptom of pancreatitis is pain felt in the upper left side or middle of the abdomen c. The patient should avoid smoking, alcoholic drinks, and fatty foods after an acute attack d. All of the above

d. All of the above

Motility disorders usually result from disruptions in the contraction and relaxation of the GI tract through all of the following mechanisms EXCEPT: a. Sympathetic innvervations b. Parasympathetic innvervations c. Intrinsic innerventions (Auerbach plexus or myenteric plexus) d. All of the above

d. All of the above

Patient was born two days ago and is presenting with difficulty feeding, distended bowels and emesis. Which of the following would be your likely diagnosis? a. Hirschsprung's disease b. Meckel's diverticulum c. Duodenal obstruction d. All of the above answers are likely diagnosis

d. All of the above answers are likely diagnosis

Premature rupture of membranes (PROM) can cause which of the following complications: a. preterm labor c. neonate sepsis b. chorioamnionitis d. All of the following

d. All of the following

Which of the following is a common site for gestational trophoblastic disease (GTD) to metastasize? a. Brain c. Genital Tract b. Liver d. All the above

d. All the above

A patient presents to your office complaining of dropping on both sides of his face and difficulty chewing. Upon examination you notice the patient has ptosis. You suspect Myesthenia Gravis and you give him some Edrophonium. His eyes shoot open and your diagnosis is confirmed. What is the most likely cause of this patient's MG? a. Infection c. Neurological injury b. Immune suppression d. Autoimmune disease/ idiopathic

d. Autoimmune disease/ idiopathic

A 70-year-old male patient presents to your office complaining of aches and stiffness in his shoulders, hips, and thighs in the mornings or after sitting for an extended period of time. Upon examination you notice the patient has limited range of motion. You also notice a mild fever and he appears fatigued. You do some blood work and find an elevated ESR. What is your most likely diagnosis? a. Muscular dystrophy c. Polymyalgia rheumatica b. MS d. Both B and C are correct

d. Both B and C are correct

A barium enema is performed on a patient and it shows an apple core lesion. What does this most likely suggest? a. Ulcerative Colitis c. Rectal prolapse b. Anal fissure d. Colon cancer

d. Colon cancer

What is hemophilia B also known as? a. Factor X deficiency c. Factor VIII deficiency b. Factor XII deficiency d. Factor IX deficiency

d. Factor IX deficiency

Uterine prolapse occurs when pelvic floor muscles and ligaments stretch and weaken. Risk factors for uterine prolapse include all of the following EXCEPT: a. One or more vaginal deliveries c. Increasing age b. Giving birth to large baby d. Nulliparity

d. Nulliparity

A mother brings her 1 month old baby into the ED where you are working. The child has a rapid heart rate and is crying with breathlessness. The mother states that her child was born prematurely and that she had a Rubella infection while pregnant. She informs you that she has had problems getting her child to gain weight and that he tires easily. You note a heart murmur on auscultation and order an echocardiogram. What is the likely diagnosis of your patient? a. Pneumonia b. Rubella c. Meningitis d. Patent Ductus Arteriosus

d. Patent Ductus Arteriosus

Which of the following would be included in the Differential Diagnosis of Factor XI deficiency disorder? a. Isolated prolonged PT time c. Documented Howell Jolly bodies b. Warfarin contamination d. Platelet function disorder

d. Platelet function disorder

A 32 y/o woman presents with for a routine physical examination. She feels well with no specific complaints. Her blood pressure is 152/92 mm Hg. You note slight fullness to the abdomen on palpation without tenderness or obvious mass. Routine labs are ordered with the following results: BUN 12, Creatinine 0.8, UA and sediment analysis: 2+ blood, trace protein, negative leukocyte esterase, negative nitrite, 10 to 20 RBCs per high power field, no leukocytes, bacteria, or other cells; rare granular cast. What is the most likely cause of the hematuria? a. Urinary tract infection c. Renal calculi b. Glomerulonephritis d. Polycystic kidney disease

d. Polycystic kidney disease

Which of the following is the most common location of bulging or herniation for a herniated disc? a. Anterior c. Posterior b. Anterior lateral d. Posterior lateral

d. Posterior lateral

A 25-year-old patient presents to your OBGYN office with concerns that she hasn't had her period in 2 months. Upon entering the room you find a young, healthy appearing woman. Her medication list is positive only for occasional NSAID use. She is in a monogamous relationship x 3 years. Her and her boyfriend use natural family planning and therefore do not have intercourse on days that her calendar says she is ovulating. Her aunt was just diagnosed with ovarian cancer so she is concerned that this could be the cause of her lack of menstruation. She is also concerned because she has been feeling nauseous, a symptom her aunt had before her diagnosis. This is considered a secondary amenorrhea, since the patient has had her period regularly in the past. You know that the most common cause of secondary amenorrhea and the most likely cause of this patient's lack of period is? a. Malignancy c. Menopause b. Low body weight d. Pregnancy

d. Pregnancy

A 30-year-old female patient presents to the ER with fever, CVA tenderness, and vomiting x 12 hours. Her last menstrual period was 2 weeks ago. She is in a monogamous relationship x 3 years. Upon entering the room, the patient appears ill. When eliciting her history you learn that she was treated for a UTI 3 weeks ago but forgot to finish her antibiotics. Your most likely diagnosis is? a. Chlamydia c. Pelvic inflammatory disease b. Tubal pregnancy d. Pyelonephritis

d. Pyelonephritis

A 40-year-old male patient presents to your clinic with complaints of knee pain. Upon examination the patient's right knee is red, warm, and inflamed. His WBCs are elevated. A bone biopsy reveals a diagnosis of acute osteomyelitis. What is the most common cause of the infection? a. Streptococcus c. Chlamydia b. Gonococcal d. Staphylococci

d. Staphylococci

A 25 years old white woman present to your clinic with a complaint of severe diarrhea for the past 6 days accompanied with abdominal pain, bloating, gas, fatigue, joint pain and headache; she has felt so bad that is afraid to eat and has lost some weight. On history you find out that this is not the first time but has happened before twice especially after a meal rich in grains. On the physical exam you find: pallor, low skin tugor, a blistery skin rash over the knees, tingling and numbness in the extremities and problems with balance; in the abdominal area you find tenderness over all quadrants especially upper quadrants. You suspect the patient may be having a reaction to "something" present in some foods so you order several exams to confirm your diagnosis. In the meanwhile you recommend your patient to: a. Stay away from milk or milk products b. Take OTC anti-diarrheals and go home and wait for results c. Hydrate well, and let it pass (probably it's just a virus) d. Stay away from food that may contain wheat, barley, oats and rye

d. Stay away from food that may contain wheat, barley, oats and rye

A 32 year old male comes in with pain over the distal bicep muscle and tendon. Patient states he was repetitively flexing his elbow this weekend when he lifting weights. According to the patient's history, what is his injury? a. anterior capsule strain c. posterolateral roaterory instability b. pronator syndrome d. biceps tendinosis

d. biceps tendinosis

G6PD deficiency? a. is an autosomal dominant disorder b. is a rare form of hemolytic anemia c. results in hepatomegaly d. may be precipitated by antibiotics e. is typically diagnosed within the first decade of life

d. may be precipitated by antibiotics

All of the following conditions can lead to chronic renal failure except: a. Diabetes d Enlarged prostate b. Hypertension c. Glomerulonephritis e. All of the above

e. All of the above

Benign prostatic hyperplasia (BPH) can cause chronic bladder outlet obstruction which may lead to what? a. Urinary retention d. Hematuria b. Renal insufficiency c. Recurrent UTI's e. All of the above

e. All of the above

Causes of spinal stenosis include which of the following? a. bone over growth d. spinal injury b. Herniated Disc c. Thickened ligaments e. All of the above

e. All of the above

Lactose intolerance usually presents with what symptoms? a. Abdominal fullness d. Flatulence b. Nausea c. Diarrhea e. All of the above

e. All of the above

Lung cancer can be caused by? a. Smoking d. family history of lung cancer b. exposure to radon gas c. exposure to asbestos e. All of the above

e. All of the above

What is a cause of urge incontinence? a. Bladder cancer d. Recent childbirth b. Infection c. Nerve injury e. All of the above

e. All of the above

Which of the following strains can cause bacterial meningitis? a. Streptococcus pneumoniae d. Listeria monocytogenes b. Neisseria meningitidis c. Haemophilus influenzae e. All of the above

e. All of the above

A fracture to the knee can include all EXCEPT? a. fractures of the patella d. tibial plateau b. femoral condyles c. tibial tuberosity e. All of the above are possible

e. All of the above are possible

Which of the following is not a cause of secondary hypertension? a. Pheochromocytoma d. Hyperthyroidism b. Aldosteronism e. CHF c. Renal Artery Stenosis

e. CHF


Ensembles d'études connexes

Pluto and dwarf planets study questions

View Set

Stress, Lifestyle, and Health OpenStax Chapter 14

View Set

Pharm Exam #2 Analgesic, Neurology, & Oncology/Hematology

View Set

Unit 19: Types of Investment Risks

View Set

Chapter 17: Ricci, Kyle, & Carter Labor and Delivery

View Set

Cardiovascular, Hematologic, and Lymphatic Systems Level 1

View Set